unit 3

¡Supera tus tareas y exámenes ahora con Quizwiz!

How is the nurse functioning when administering a drug that has PRN as part of the prescription? 1. Collegially 2. Dependently 3. Independently 4. Interdependently

1. Collegial or collaborative interventions are actions the nurse performs in conjunction with other health-care team members. 2. Dependent interventions are those activities performed under a practitioner's direction and supervision. 3. Independent interventions are those activities the nurse is licensed to initiate based on knowledge and expertise. 4. CORRECT: An interdependent intervention requires a practitioner's order associated with a set parameter. The parameter, whenever necessary, requires that the nurse use judgment in implementing the order

A client with HIV has no health insurance. The single-dose medication is $1100 per month, and the client is unable to obtain this medicine. Instead, the healthcare clinic will provide three medicines that must be taken four times a day. The nurse will: 1.Advocate for the client's rights, since it is illegal to deny the single-dose medication. 2.Recognize that this is an ethical issue and encourage the client to discuss his feelings. 3.Report this incident to the ethics board. 4.Call the pharmaceutical company and demand free pills for the client.

2

A nurse who reports suspected child abuse, honestly believing it to have occurred, is not subject to civil or criminal liabilities when the subsequent investigation does not make a determination of abuse. This is called A) good faith immunity. B) protection of privacy. C) breach of confidentiality. D) criminal malfeasance.

A

Nurses agree to be advocates for their patients. The practice of advocacy calls for the nurse to which of the following? A. Assess the patient's perspective and explain it when necessary. B. Apply the law to the patient's clinical condition. C. Seek out the nursing supervisor to resolve conflict. D. Document all clinical changes in the medical record every 2 hours.

A.

A medication error occurred and the nurse is preparing to complete an incident report. Which information is required to thoroughly complete this report? Select all that apply. A) Name of client involved in the incident B) Location where incident report is completed C) Date and time of the incident D) Medication involved in the incident E) Number of hours the nurse was at work before the incident occurred

ACD

A nurse who reports suspected child abuse, honestly believing it to have occurred, is not subject to civil or criminal liabilities when the subsequent investigation does not make a determination of abuse. This is called A) good faith immunity. B) protection of privacy. C) breach of confidentiality. D) criminal malfeasance.

Answer: A Explanation: A) In every state, healthcare workers are protected from civil or criminal liabilities when they report suspected child abuse in good faith, even if the subsequent investigation does not make a determination of abuse. This is called good faith immunity. This is not protection of privacy, breach of confidentiality, or criminal malfeasance.

The nurse is caring for a client in the intensive care unit (ICU) who was in a motor vehicle crash. The healthcare provider asks the nurse to extubate the client because there is no communication between the brain and body due to a cervical fracture. The family agrees with the decision of the healthcare provider, but the nurse is uncomfortable pulling the tube. Which is the reason the nurse is experiencing difficulty with this task? A) An ethical conflict B) Personal values C) Legal issues D) Cultural values

Answer: B Explanation: A) The nurse is distressed because of personal values, which are in conflict with causing the client's death. The decision is within ethical principles. Cultural values are not in evidence in this instance. Extubating this client would not be a legal decision.

The nurse is discussing the importance of incident reports with colleagues. Which explanation should the nurse include? A. "Incident reports let lawyers know that nurses care about patient care." B. "Incident reports can protect the hospital in case of future patient care events." C. "Incident reports withhold information about mistakes in patient care." D. "Incident reports can be used to improve the delivery of future patient care."

Answer: D Completing incident reports is worthwhile because they can be used to improve the delivery of future patient care. Their purpose is not to protect the hospital, to inform lawyers, or to withhold information about mistakes.

The nurse is in the midst of a complicated client care situation and is not sure what needs to be done with some information. Which healthcare issues must the nurse report to the state? Select all that apply. A) Amputation of a limb B) Death of a client C) Death of a neonate D) Diagnosis of tuberculosis E) Kidney transplant

B C D

Which of the following is the best example of a situation where a nurse could demonstrate workforce advocacy? A. A client wishes to become an organ donor in spite of family opposition. B. A client feels pressured by a health care provider into agreeing to a specific medical procedure. C. A nurse witnessing a patient signing a formed consent at their own will

B.

Which nursing action best complies with the expectations for nursing care defined by the "never events" identified by the 2012 Rules of Participation for Hospitals? A. Requiring all unlicensed nursing personal to attend shift reports B. Providing care when convenient for the client whenever possible C. Attending an in-service on evidence-based practice on urinary catheter care D. Reporting suspected elder abuse to the nursing manager immediately

C Nurses are required to develop greater expertise in the provision of evidence-based patient care, case management, and discharge planning in order to avoid "never events." Although the other options are appropriate nursing actions, they are not related to "never events"; hospital-acquired conditions are considered reasonably preventable.

The nurse preceptor tells the nursing student that participation in nursing research will most likely help the student: Improve daily practice. Develop a better attitude about work. Improve communication with clients. Improve the development of skills.

Improve daily practice.

A hospital donates medical supplies for a disaster relief initiative. Which approach was used to make this decision? a. Pluralist b. Utilitarian c. Contractarian d. Deontological

b. Utilitarian

The nurse is implementing an ordered bowel preparation for a patient who is scheduled for a colonoscopy. Which is the most serious consequence that is prevented by an effective bowel preparation? 1. Discomfort 2. Misdiagnosis 3. Wasted expense 4. Psychological stress

1. Although this may occur, it is not the most serious outcome of an inappropriate preparation for a colonoscopy. 2. CORRECT: Fecal material in the intestines can interfere with the visualization, collection, and analysis of data obtained through a colonoscopy, resulting in diagnostic errors. 3. A test may have to be cancelled or performed a second time if the patient has an ineffective bowel preparation. Although this is a serious consequence, it is not life threatening. 4. Although this is a serious consequence, it is not life threatening.

The nurse changes a patient's dry sterile dressing. How is the nurse functioning when performing this task? 1. Interdependently 2. Collaboratively 3. Independently 4. Dependently

1. The changing of a dry sterile dressing is an interdependent action by the nurse when the practitioner's order for wound care states: Dry Sterile Dressing PRN. 2. In this situation, the nurse is not working with other health-care professionals to implement a practitioner's order. 3. This intervention is not within the scope of nursing practice without a practitioner's order. 4. CORRECT: A nurse is not permitted legally to prescribe wound care. The nurse needs a practitioner's order to provide wound care.

A medication error occurred and the nurse is preparing to complete an incident report. Which information is required to thoroughly complete this report? Select all that apply. A) Name of client involved in the incident B) Location where incident report is completed C) Date and time of the incident D) Medication involved in the incident E) Number of hours the nurse was at work before the incident occurred

A C D

Which of the following actions is required of the nurse practicing advocacy? Select all that apply. A. Speak up for patient care issues even when others may disagree B. Contribute money towards patient healthcare costs because the patient is unable to pay C. Assess the patients point of view and be able to articulate it D. Document all clinical changes in the patients medical record in a timely and legible manner E. Become an active member of professional nursing organizations

A, C, D

One of the elements of professional negligence is the failure to act according to the standard of care or breach of duty. Standard of care may best be defined as which of the following? (Select all that apply.) A. Nursing competence as defined by the State Nurse Practice Act B. Giving nursing care in the most expedient and timely way possible C. The degree of nursing judgment and skill given by a reasonably prudent nurse under similar circumstances D. Providing health services according to community expectations and ordinances

A, C, D Duty under any standard of care is the degree of nursing judgment and skill given by a reasonably prudent professional in the same or similar circumstance. A standard of care is defined by the State Board of Nursing Practice Act and current community standards. Expedient care, although timely, may not necessarily meet the standard of care.

Two nursing students are assigned to work together to create a care plan. One of the students comes to class the following day with a fully completed care plan and tells their partner that their sister took this class 3 years earlier and they have just copied the sister's work. Which ethical dilemma does this represent? Professional values Academic dishonesty Conflicting loyalties and obligations Allocation of limited resources

Academic dishonesty

1) The nurse is caring for a terminally ill child. The parents have decided to remove their child from life support. This decision was met with much opposition from other nurses on the unit. Which action by the nurse displays the role of advocate? A) Respecting the parents' decision B) Telling the parents they are making the right decision C) Asking to be assigned to a different client D) Referring the parents to social services

Answer: A Explanation: A) The nurse best advocates for the family by supporting the family's right to make this decision. Telling the clients they are making the right decision is inappropriate and does not support advocacy. Referring the parents to another entity points to feelings of unease about the parents' choice. Asking to be assigned to another client does not honor the right of clients and families to make decisions about health care.

During an assessment of a child in the urgent care clinic, the nurse notes that the child has a swollen and split lip. When asking the parent how the child's lip injury occurred, the parent responds, "We are here for my child's ear not my child's lip." Which is the rationale for reporting this incident? A) The child reports that a parent caused the injury. B) The lip injury is unrelated to the ear infection. C) The nurse can be sued if there is no abuse. D) Suspected abuse must be reported.

Answer: D Explanation: A) Suspected child abuse must be reported by law. Healthcare personnel are protected by good faith immunity because the ultimate goal is the protection of the child. The lip injury being unrelated to the ear infection is not a reason to report the injury. Most children will not accuse an abuser; rather, they generally protect the abuser. The reason for the law is that experts can assess the situation and determine if abuse has occurred. The nurse is protected by good faith immunity.

The nurse observes a healthcare provider discussing an operative procedure with a client and determines that informed consent was achieved. Which information was included in the informed consent process? Select all that apply. A) The provider's disapproval if the surgery is not performed B) The health problem that requires surgery C) The purpose of the surgery D) The expectations of the surgery E) Outcome if surgery is not performed

B C D E

A client is receiving care in the hospital for life-threatening injuries sustained in a motor vehicle crash and is taken immediately to surgery. There is no family available to provide consent; however, the client's medical record is available and reviewed by the nurse. Which treatments are inappropriate in this situation? Select all that apply. A) Emergency surgery B) Treatment that was previously refused C) Treatment that violates religious beliefs D) Medications to treat the injury E) Experimental medications for a research study

B C E

Which of the following shows the best example of patient advocacy? A. A client wishes to be an organ donor despite families disapproval. B. A patient feels pressured by their provider to accept a medical procedure. C. Influencing the patient to make a decision based on your past experience. D. Trying to get a patient to take a medication after refusal.

B.

A student nurse administers a medication to the wrong client while the instructor is with another student. Which statement by the instructor is most appropriate in this situation? A) "You have placed the nursing student program in danger." B) "You may be sued by the hospital for the extra care cost to the client." C) "You are expected to practice like a licensed nurse." D) "You have set a bad example for the other students."

C

Although you normally work in a hospital setting, you have volunteered at a homeless shelter at a blood pressure clinic. If an incident occurs at the blood pressure clinic, what is your most likely liability protection provider? A. Your employer hospital malpractice insurance B. Your home insurance C. Your professional liability insurance D. No one (There is a small likelihood that a nurse will be sued in this type of situation.)

C Your employer is only obligated to defend your actions as an agent of the employer when you are working within the scope of your employment. Your home insurance does not cover your actions in this situation unless you also have professional liability attached to your homeowner's policy.

Which of the following statements describes the nurse's duty to investigate suspected abuse of a pediatric client before reporting it? A) The nurse must question a parent or guardian about the suspected abuse. B) The nurse must personally observe the client being abused. C) The nurse must identify at least two witnesses who will testify that the client was abused. D) The nurse does not need to investigate suspected abuse of a pediatric client.

D

A woman who is 6 months pregnant is involved in a motor vehicle crash and sustains massive head injuries. The patient is kept alive on a ventilator until the baby can be delivered 2 months later. Which factor describes the ethical dilemma posed by this situation? The mother's right to patient autonomy versus the fetus's survival Lack of family decision making in this process The mother's obvious desire for the baby to be born verses the baby's being born early Whether it was ethical to keep the mother alive using medical technology

The mother's right to patient autonomy versus the fetus's survival

Which action can aid the nurse in making ethical decisions in patient care? Reviewing and revising policies as needed Using evidence-based nursing protocols Taking the most conservative course of action Basing decisions on what most colleagues believe

Using evidence-based nursing protocols

Because of a traffic fatality, a donor heart becomes available. With so many people awaiting a heart transplant, the doctor at the regional transplant center needs to consider the key criteria for distributing the donor heart. Which two criteria should the doctor consider? Choose 2 answers a. Ability to pay b. Medical need c. Age of patient d. Time on waiting list

b. Medical need d. Time on waiting list

An unconscious elderly patient in acute respiratory failure is brought to an emergency department and is intubated. The patient's family requests that the patient be extubated in keeping with the patient's advance directive. The physician refuses, saying it would be detrimental to the patient's health. Which patient right has been violated? a. The right to fidelity b. The right to veracity c. The right to autonomy d. The right to informed consent

c. The right to autonomy

A nurse confronts a patient stealing a vial of narcotics from a medicine cart, and the patient tearfully requests the nurse to take no action. Which action represents appropriate professional behavior of the nurse? a. Report it to the police b. Maintain confidentiality c. Honor the patient's wishes d. Report it to the nurse in charge

d. Report it to the nurse in charge

State legislatures are responsible for: 1. Standardized care plans 2. Enactment of Nurse Practice Acts 3. Accreditation of educational nursing programs 4. Certification in specialty areas of nursing practice

1. Nursing team members or an interdisciplinary team of health-care providers write standardized care plans. 2. CORRECT: Every state has its own Nurse Practice Act that describes and defines the legal boundaries of nursing practice within the state. 3. The National League for Nursing Accrediting Commission, the Commission on Collegiate Nursing Education, and state education departments are the major organizations accrediting nursing education programs in the United States. 4. The American Nurses Association and other specialty organizations offer certification in specialty areas in nursing practice.

Nursing practice is influenced by the doctrine of respondeat superior. What is the basic concept related to this theory of liability? 1. Nurses must respond to the Supreme Court when they commit acts of malpractice 2. Health-care facilities are responsible for the negligent actions of the nurses whom they employ 3. Nurses are responsible for their actions when they have contractual relationships with patients 4. The laws absolve nurses from being sued for negligence if they provide inappropriate care at the scene of an accident

1. This is unrelated to respondeat superior. Negligence and malpractice, which are unintentional torts, are litigated in local courts by civil actions between individuals. 2. CORRECT: The ancient legal doctrine respondeat superior means "let the master answer." By virtue of the employer-employee relationship, the employer is responsible for the conduct of its employees. 3. Individual responsibility is unrelated to respondeat superior. A nurse can have an independent contractual relationship with a patient. When a nurse works for an agency, the contract between the nurse and patient is implied. In both instances the nurse is responsible for the care provided. 4. This is unrelated to respondeat superior. Good Samaritan laws do not provide absolute immunity.

In order to best ensure client safety, which of the following questions should a staff nurse ask first when determining whether to accept a client assignment that is made when several staff members have reported off sick? A. "Do I have the experience and knowledge to care for these patients?" B. "Is this a staffing crisis or a result of typical staffing patterns?" C. "Can I expect to be mandated to stay an additional shift?" D. "How long will it be before I get additional help?"

A Although all these questions can help the staff nurse think critically about accepting the assignment, initially the important question is whether the nurse possesses the experience, skills, and knowledge necessary to safely provide appropriate care for the clients she is asked to care for.

A nurse educator is planning a class for a group of nursing students regarding risk management. Which information should the educator include in this presentation? Select all that apply. A) Risk management seeks to prevent harm. B) Risk management empowers clients. C) Risk management controls the cost of supplies. D) Risk management examines past mistakes and identifies potential hazards. E) Risk management ensures that nurses are truthful.

A D

c (Rationale The item the nurse would circle as being false is that if​ follow-up treatment was​ needed, the chart should not record that fact.​ Legally, the chart must record the delivery of​ follow-up treatment. It is true that most incidents do not result in lawsuits. It is true that incident report should be written as though discoverable. It is true that if an incident report was​ written, the chart should not record that fact.)

A nurse recently attended the​ hospital's mandatory annual education series about filling out incident reports. At the​ end, knowledge was tested using a​ True-False posttest. Which item would the nurse circle as being​ false? a If an incident report was​ written, the chart should not record that fact. b Most incidents do not result in the filing of a civil lawsuit in court. c If​ follow-up treatment was​ needed, the chart should not record that fact. d Incident reports should be written as though discoverable.

A nurse is caring for a client who decides not to have surgery despite significant blockages in his coronary arteries. The nurse understands that this client's choice is an example of which of the following: A. Fidelity B. Autonomy C. Justice D. nonmaleficence

A: INCORRECT: Fidelity is an agreement to keep promises. The nurse has not made any promises; this is the client's decision. B: CORRECT: In this situation, the client is exercising his right to make his own personal decision about surgery, regardless of others' opinions of what is "best" for him. This is an example of autonomy. C: INCORRECT: Justice is fairness in care delivery and in the use of resources. Because the client has chosen not to use them, this principle does not apply. D: INCORRECT: Nonmaleficence is the avoidance of harm or injury. In this situation, harm can offur whether ot not the client has surgery. However, because he chooses not to, this principle does not apply.

A nurse educator is planning a class for a group of nursing students regarding risk management. Which information should the educator include in this presentation? Select all that apply. A) Risk management seeks to prevent harm. B) Risk management empowers clients. C) Risk management controls the cost of supplies. D) Risk management examines past mistakes and identifies potential hazards. E) Risk management ensures that nurses are truthful.

AD

A health care provider has written an order for a patient to receive a medication every 6 hours for 7 days. You note that the patient has indicated that she is allergic to this medication (rash, shortness of breath). Which of the following should you do first? A. Contact the health care provider. B. Contact the pharmacist. C. Place a "hold" note on the medication administration record (MAR). D. Contact the nursing supervisor.

C You must stop any possible administration of the medication that will cause an allergic response in the patient. Placing a hold order on the MAR stops anyone from inadvertently administering the medication. You should then contact the pharmacy and the health care provider. The nursing supervisor should be contacted if the health care provider does not act to rescind the order.

A 14 years-old girl newly diagnosed with diabetes is preparing for discharge. Which of the following activities BEST describes the nurse's role as a client advocate? A. Arranging for a visit with a home health nurse B. Providing written medication instructions to the client's parents C. Instructing the client to follow up with her provider in 4 weeks D. Teaching the client how to administer insulin injection

D.

A patient is soon to be pronounced brain dead, and the family has been approached about organ donation. When the family returns the next day, the son mentions reading about organ donation on the internet and learning that hospitals charge the families of donors for the organ donation. The son announces that the family has decided to refuse organ donation. Which course of action by the nurse is most ethical? Informing the family that this information is completely untrue and that the family must be careful of what they see on the internet Providing the family with information from the organ procurement agency and asking that a representative of the agency speak with the family Documenting the family's decision that organ donation will be refused Trying to talk the family into reconsidering because so many donor organs are needed

Providing the family with information from the organ procurement agency and asking that a representative of the agency speak with the family

A patient requests the withdrawal of hemodialysis, consistent with his advance directive. The physician, however, insists on disregarding the patient's wishes. Which ethical right of the patient pertains to this scenario? a. Justice b. Authority c. Autonomy d. Informed consent

c. Autonomy

A patient comes to a healthcare clinic and is HIV positive. A nurse with an immune disorder refuses to participate in the patient's care. Why may the nurse refuse to care for the patient? a. Personnel policies allow refusal. b. Americans with Disabilities Act (ADA) regulations protect the nurse. c. HIV patients are excluded from treatment in public clinics. d. Potential harm to the nurse outweighs the gain for the patient.

d. Potential harm to the nurse outweighs the gain for the patient.

The nurse is approached by the parents of a teen admitted with a uterine hemorrhage. The father asks, "Is my daughter pregnant?" Which response by the nurse is appropriate? "You can ask the admitting physician. I'm sure she'll discuss your daughter's condition with you." "I know that this situation must be hard for you to deal with. I'll let you know when I find out." "I cannot discuss her medical condition with you. All of our patients have privacy rights." "I don't think you have anything to worry about. She's a lovely young woman."

"I cannot discuss her medical condition with you. All of our patients have privacy rights."

Licensure of Registered Professional Nurses is required primarily to protect: 1. Nurses 2. Patients 3. Common law 4. Health-care agencies

1. Licensure does not protect the nurse. Licensure grants an individual the legal right to practice as a Registered Nurse. 2. CORRECT: Licensure indicates that a person has met minimal standards of competency, thus protecting the public's safety. 3. Licensure does not protect common law. Common law comprises standards and rules based on the principles established in prior judicial decisions. 4. Licensure does not protect health-care agencies. The Joint Commission determines if agencies meet minimal standards of health-care delivery, thus protecting the public.

The nurse is concerned about being sued for negligence when providing care. Which nursing actions may be grounds for negligence? Select all that apply. A) Client fell getting out of bed because the call light was not used. B) Client name band was checked prior to providing all medications. C) Client's morning medications were administered in the early afternoon. D) Client states not understanding activity restrictions and wound eviscerated. E) Client documentation did not include appearance of infiltrated IV site.

A C D E

The nursing instructor is evaluating the success of training provided to staff nurses on ways to reduce the incidence of pediatric medication errors. Which observations indicate that training has been effective? Select all that apply. A) Staff nurses are double-checking medication calculations. B) Staff nurses are refusing to dilute medications. C) Staff nurses are using liquid preparations. D) Staff nurses are asking the pharmacy to prepare the exact doses. E) Staff nurses are asking each other to validate placement of decimal points.

A C E

A nurse offers pain medication to a client who is postoperative prior to ambulation. The nurse understands that this aspect of care delivery is an example of which of the following ethical principles: A. Fidelity B. Autonomy C. Justice D. Beneficence

A: INCORRECT: Fidelity is an agreement to keep promises. Unless the nurse has specifically promised the client a pain-free recovery, which is unlikely, this principle does not apply to this action. B: INCORRECT: Autonomy is the right to make personal decisions, even when they are not necessarily in the person's best interest. In this situation, the nurse is delivering responsible client care. This principle does not apply. C: INCORRECT: Justice is fairness in care delivery and in the use of resources. Pain management is available for all client's who are postoperative, so this principle does not apply. D: CORRECT: Beneficence is taking positive actions to help others. By administering pain medication before the client attempts a potentially painful exercise like ambulation, the nurse is taking a specific and positive action to help the patient.

The nursing instructor is evaluating the success of training provided to staff nurses on ways to reduce the incidence of pediatric medication errors. Which observations indicate that training has been effective? Select all that apply. A) Staff nurses are double-checking medication calculations. B) Staff nurses are refusing to dilute medications. C) Staff nurses are using liquid preparations. D) Staff nurses are asking the pharmacy to prepare the exact doses. E) Staff nurses are asking each other to validate placement of decimal points.

ACE

The home health nurse suspects that another nurse providing home care to a client has been taking the client's narcotics. Which action should the nurse carry out? A) Follow the reporting procedures for her agency. B) Tell the client to confront the other nurse. C) Confront the other nurse about the suspected theft. D) Have the client file a police report.

Answer: A Explanation: A) Nurses have a legal responsibility to report any professional whom they suspect of engaging in illegal, immoral, or unethical activities. Normally, the nurse making such a report will do so following established procedures at the facility at which the nurse is employed. Both state nurse practice acts and the ANA Code of Ethics require nurses to report unethical nurse behaviors, including boundary violations. The nurse should not leave the responsibility of addressing this problem to the client or directly confront the other nurse.

The nurse is caring for a terminally ill pediatric client. The parents have decided to remove their child from life support. Which action by the nurse displays the role of client advocate? A) Respecting the parents' decision B) Telling the parents they are making the right decision C) Asking to be assigned to a different client D) Referring the parents to social services

Answer: A Explanation: A) The nurse best advocates for the family by supporting the family's right to make this decision. Telling the clients they are making the right decision is inappropriate and does not support advocacy. Referring the parents to another entity points to feelings of unease about the parents' choice. Asking to be assigned to another client does not honor the right of clients and families to make decisions about healthcare.

Which action demonstrates correct reporting of suspected child abuse? A) The nurse includes the entirety of the client's medical record. B) The nurse compiles a report with all pertinent information that is factually true. C) The nurse recommends that the organization report the abuse to state authorities. D) The nurse reports only information the client has authorized for release.

Answer: B Explanation: A) Reports should be complete and accurate and should be made according to the policy of the organization for which the nurse works. In addition to reporting the abuse within the organizational framework, the nurse should personally report the abuse to the proper authorities. When abuse is reported, all pertinent information in the client's medical record (not simply the entire record) is required by law to be disclosed to the reporting agency. As such, reporting abuse or suspected abuse represents an exception to client confidentiality rules.

A novice nurse attends a lecture regarding risk management. Which action should the nurse implement to reduce risks in practice? A) Not discussing errors made B) Questioning every order that the physician writes C) Urging the nurse's organization to purchase liability insurance D) Storing unused equipment in the halls of the unit

C

In what situation may a nurse deliver care that is not considered at the level of required standard of care? A. When directly ordered to do so by a physician B. When the nursing area is considered understaffed C. When the situation is determined to be an emergency D. When the client refuses care that would meet the standard of care.

C Nursing care rendered in a life-threatening emergency may breach the standard of care required under ordinary circumstances. The nurse may not knowingly provide substandard care even when ordered to do so by medical staff. Understaffing is not an acceptable reason for substandard care nor is a client's refusal to consent.

The nurse in an inpatient hospice realizes that part of hospice care is aggressive pain management, including the administration of high doses of pain medications to patients. The nurse is not morally opposed to this practice, believing that it gives comfort in a patient's final days. Which process does the nurse apply to reach this decision? Social justice Values clarification Continuing education Patient advocacy

Values clarification

d

What does the legal term​ "discovery" mean? a Process of putting a witness on the stand to testify b Process of finding out the individual to blame for the unfortunate event c Process of filing of a civil lawsuit to recover punitive damages d Process of obtaining information before a trial

c,d,e (An incident report includes the​ location, date, and time of the event. It often includes the​ client's account of the event in quotes. The client​'s home address and health insurance are not part of an incident report.)

Which elements are commonly found in an incident​ report? ​(Select all that​ apply.) ​a Client's health insurance b ​Client's home address c Location of event ​d Client's account of event e Date and time of event

A patient is considering participating in a cancer research study that is being conducted by a government organization. Prior to enrollment, the patient is concerned about termination if the employer finds out about the patient's participation. Which right under informed consent should be discussed with this patient to alleviate any concerns? a. A right to confidentiality b. A right to continued employment c. A right to protection under HIPAA d. A right to job security under the ADA

a. A right to confidentiality

A nurse discusses a patient's diabetic meal planning on discharge. Sample meal planning includes listing foods that are used at home for diet compliance. Which patient-centered care value is the nurse demonstrating? a. Engagement b. Companionship c. Emotional support d. Personal care support

a. Engagement

A research hospital is conducting clinical trials for a new medical procedure, and several of its affiliated surgeons are part of the research team. One surgeon has been actively recruiting patients from his colleagues to participate. What is the primary requirement for this recruitment? a. The patients have given a written consent for the research. b. An adequate number of patients are recruited to ensure proficiency. c. The patient sample is adequately mixed in demographic characteristics. d. The patients have no contraindicated pain to confuse the research results.

a. The patients have given a written consent for the research.

A middle-aged woman comes into a local emergency room complaining of fatigue. The patient notes a heavy monthly menstrual cycle. After careful examination and review, the nurse decides the patient will need a blood transfusion, however, the patient's religious preference does not allow for transfusions. The patient refuses the blood transfusion. What should the nurse do in this situation? a. Transfuse the patient b. Honor the patient's wishes c. Encourage the patient to accept the transfusion d. Discuss the patient's wishes with the next of kin

b. Honor the patient's wishes

A doctor persuades a patient to become part of a clinical trial. The doctor intends to have the patient sign an informed consent form. How should the doctor ensure that ethical procedures are being followed in the research? a. Advise the patient that insurance coverage applies to this study b. Inform the patient that authorization applies throughout the study c. Require the patient to discuss the study with family before enrolling d. Allow the patient to choose what vulnerable population is appropriate

b. Inform the patient that authorization applies throughout the study

Security staff were called to restrain a psychiatric patient in a hospital's behavioral medicine unit. The security staff were not accustomed to responding to the unit, leading to trauma that nearly made the patient die. Which Joint Commission policy will assist the hospital in this situation? a. Confidentiality policy b. Sentinel event policy c. Treatment interventions policy d. First responders training policy

b. Sentinel event policy

A patient with tuberculosis was admitted to the hospital and is refusing treatment. When the patient's nurse leaves the room, the patient leaves the hospital. Which ethical duty should the nurse perform? a. Treat b. Warn c. Maintain privacy d. Maintain confidentiality

b. Warn

A patient signs an informed consent for an experimental trial. After three months of participating in the study, the patient does not want further doses of the medication. Which right is the patient exercising? a. Privacy b. Withdrawal c. Non-coercion d. Independence

b. Withdrawal

The client asks the​ nurse, "Why am I receiving this​ medication?" Which action should the nurse take if not familiar with the​ medication? A. Instructing the client to tell the nurse the reason why the client thinks the medication is being given B. Holding the medication and documenting that the client refused C. Looking up information on the medication and telling the client the reason it is being given D. Leaving the​ client's oral medication at the bedside and allowing the client to decide whether to take it

c

An 86-year-old single adult with three children and a living will is hospitalized with pneumonia. Despite antibiotics and supportive care, the patient's pneumonia progresses, and hypoxemia and delirium develop. The patient's physician recommends mechanical ventilation. Which concept of autonomy applies to this scenario? a. Justice b. Beneficence c. Advance directives d. Right to refuse treatment

c. Advance directives

A diabetic patient was going to eat a cream puff and take extra insulin. After being advised that this was not a good practice, the patient decided to eat the sugar-free cookies. Which two ethical principles are in conflict with each other in this medical decision? Choose 2 answers a. Justice b. Veracity c. Autonomy d. Beneficence

c. Autonomy d. Beneficence

A 70-year-old patient is admitted to the hospital because of an accidental gunshot wound. The patient is pale and losing blood and needs a blood transfusion. The patient is responsive but refuses the transfusion based on religious beliefs. Which two rights is this patient exercising in regard to care? Choose 2 answers a. Dignity b. Justice c. Capacity d. Autonomy

c. Capacity d. Autonomy

A patient in the hospital confides in a nurse about an extra-marital affair. The affair does not impact the patient's medical condition, and the patient asks the nurse to keep this information in confidence. What should the nurse do in this situation? a. Counsel the patient b. Disclose it to the spouse c. Honor the patient's wishes d. Disclose it to the hospital administrator

c. Honor the patient's wishes

A patient was admitted into the hospital with a chronic condition. The hospital transporter came to pick up the patient for a surgical screening procedure. The patient refused because he was not aware that it was being done. Which type of consent should have been executed? a. Verbal b. Implied c. Informed d. Expressed

c. Informed

An anxious patient repeatedly uses the call bell to get the nurse to come to the room. Finally the nurse says to the patient, "If you keep ringing, there will come a time I won't answer your bell." What legal term is related to this statement? 1. Slander 2. Battery 3. Assault 4. Libel

1. This is not an example of slander, which is a false spoken statement resulting in damage to a person's character or reputation. 2. This is not an example of battery, which is the unlawful touching of a person's body without consent. 3. CORRECT: This is an example of assault. Assault is a verbal attack or unlawful threat causing a fear of harm. No actual contact is necessary for a threat to be an assault. 4. This is not an example of libel, which is a false printed statement resulting in damage to a person's character or reputation.

12) The nurse providing an initial assessment informs the client of her rights. The nurse knows which value is basic to client advocacy? A) The client is a holistic, autonomous being who has the right to make choices and decisions. B) The nurse has the responsibility to ensure the client's spirituality is respected. C) The client has the right to refuse care. D) The client is a dependent being who has the right to expect the nurse to solve all healthcare needs.

Answer: A Explanation: A) Clients deserve advocates who can help them navigate healthcare bureaucracy and help them access required and needed resources. Values basic to client advocacy include: the client is a holistic, autonomous being who has the right to make choices and decisions; the client has the right to expect a nurse-client relationship that is based on shared respect, trust, collaboration in solving problems related to health and healthcare needs, and consideration of his or her thoughts and feelings; and the nurse has the responsibility to ensure the client has access to healthcare services that meet health needs.

4) The home health nurse is assigned to care for a client who requires wound care. The nurse is assessing the family's ability to change a dressing on a wound that the client cannot reach. The nurse asks the family to change the dressing while the nurse watches. What is the most appropriate response by the nurse? A) "Great job! Let me show you a technique that can make the dressing change more effective." B) "This wound will never heal if you change the dressing that way." C) "I'll come and do it every day so that the wound will heal more quickly." D) "You are not doing it right. I will just have to come every day until you learn how."

Answer: A Explanation: A) To facilitate the client and family member learning this skill, the nurse evaluates their efforts and gently helps them make some changes. This empowers the family and increases the chances of healing. Saying that the family is doing it wrong is demoralizing. Sarcasm only lowers the self-esteem of the family member. Offering to do it every day sends the message that the nurse does not trust the family's ability to learn the proper procedure.

When faced with ethical dilemmas, which are some of the elements of risk management that can assist nurses in decision making? Select all that apply. A) Education B) Peer support and consultation C) Resource accumulation D) Righteousness E) Financial support

Answer: A, B, C Explanation: A) In addition to the ANA code of conduct in ethical situations, education and didactic training represent another source for developing primary risk-management skills. A practitioner's professional network, consisting of peers, supervisors, and colleagues, can be a significant resource for primary prevention of ethical challenges. Resource accumulation involves acquiring the requisite resources and skills prior to the occurrence of a dilemma. Righteousness and financial support are not elements of risk management.

A nurse is caring for an older adult client with terminal cancer. The client's family wants to continue treatment, but the client would like to discontinue treatment and go home. The nurse agrees to be present while the client tells the family. Which principle is the nurse supporting? A) Beneficence for the client B) Autonomy for the client C) Nonmaleficence for the client D) Justice for the client

Answer: B Explanation: A) Autonomy refers to the right to make one's own decisions. The nurse is supporting this principle by supporting the client in his decision. Nonmaleficence is the duty to "do no harm." Justice is often referred to as fairness. Beneficence means "doing good."

The nurse observes a healthcare provider discussing an operative procedure with a client and determines that informed consent was achieved. Which information was included in the informed consent process? Select all that apply. A) The provider's disapproval if the surgery is not performed B) The health problem that requires surgery C) The purpose of the surgery D) The expectations of the surgery E) Outcome if surgery is not performed

Answer: B, C, D, E Explanation: A) For informed consent to be achieved, the client should receive the following information: the diagnosis or condition that requires treatment, purposes of the treatment, what the client can expect to feel and experience, intended benefits of the treatment, risks, and what could occur if the surgery is not performed or if alternatives to the treatment are chosen. To give informed consent voluntarily, the client must not be coerced in any manner. If the client provides consent due to fear of disapproval by a healthcare provider, such consent is not considered to be voluntary. Coercion of any kind invalidates the consent.

A client is receiving care in the hospital for life-threatening injuries sustained in a motor vehicle crash and is taken immediately to surgery. There is no family available to provide consent; however, the client's medical record is available and reviewed by the nurse. Which treatments are inappropriate in this situation? Select all that apply. A) Emergency surgery B) Treatment that was previously refused C) Treatment that violates religious beliefs D) Medications to treat the injury E) Experimental medications for a research study

Answer: B, C, E Explanation: A) In most states, the law assumes an individual's consent to medical treatment when the person is in imminent danger of loss of life or limb and unable to give informed consent. In other words, the emergency doctrine assumes that the individual would reasonably consent to treatment if able to do so. This doctrine serves as a guiding principle that permits healthcare providers to perform potentially life-saving procedures under circumstances that make it impossible or impractical to obtain consent. Treatment that was previously refused or violates the client's documented religious beliefs is not appropriate. Experimental medications that are being initiated in conjunction with a research study are also not appropriate.

In order to best address workplace satisfaction among its nursing staff, a hospital's administration provides: A. a yearly salary raise. B. tuition reimbursement. C. additional vacation time. D. an extra 15-minute break per shift.

B Areas that have been identified as affecting nursing satisfaction include: 1) opportunities to influence decisions about workplace organization; 2) recognition of accomplishments and work well done; 3) opportunities for professional development and advancement; and 4) opportunities to influence decisions about patient care. Tuition reimbursement would meet the interest in professional development of many nurses. Although the other options are considered favorable gestures, they do not address the identified factors.

The nurse is caring for a patient who has no brain activity and no hope of recovery. Three family members agree to withdraw care while one family member refuses this course of action stating, "I saw a woman on TV who was declared brain dead but who woke up a year later." Which indication of unclear values applies in this situation? Numerous or repeated instances of the same behavior Inconsistent communication or behavior Ignoring a healthcare professional's advice Confusion or uncertainty about which course of action to take

Confusion or uncertainty about which course of action to take

The nursing instructor is explaining​ evidence-based practice​ (EBP) to nursing students. Which statements are appropriate to include in the​ explanation?​(Select all that​ apply.) EBP considers the client​'s ​needs, values, and choices. EBP is reflective of the best evidence from current research. EBP promotes generalization of client care. EBP tests hypotheses about​ health-related conditions. EBP incorporates the nurse​'s clinical expertise.

EBP considers the client​'s ​needs, values, and choices. EBP is reflective of the best evidence from current research. EBP incorporates the nurse​'s clinical expertise.

The nurse taking a certification class becomes aware that a colleague enrolled in the same class is practicing academic dishonesty. The nurse is trying to decide what to do about the situation. Which is the first step that the nurse should take in making an ethical decision? Making a decision and sticking to it Asking another trusted student what to do Waiting until there is more evidence Identifying a range of actions with potential outcomes

Identifying a range of actions with potential outcomes

A terminally ill patient wants to remove aggressive life-sustaining measures. The patient's two children strongly disagree. What patient right must the physician consider? a. Authority b. Adequacy c. Competency d. Informed consent

c. Competency

The nurse is asked to give a medication to a client that will terminate the client's pregnancy. The nurse is opposed to this method of abortion. Which of the following strategies might help the nurse decide the correct action in this case? 1.Have self-awareness of values and beliefs. 2.Refuse to participate in the collaboration regarding this client. 3.Withhold the medication until the prescribing person arrives. 4.Tell the client that taking the medication is immoral.

1

Which hospital-wide policy would best address the nurse's negative view of job satisfaction? A. Making it possible to earn additional personal leave time based on their absentee records B. Providing a dedicated parking area close to the hospital for nursing staff C. Instituting biannual staff recognition days that include a free meal in the cafeteria D. Providing security backup when there is a perceived sense of workplace violence

A RNs are generally satisfied with their jobs when hospital administrators emphasize quality of patient care; recognize the importance of their personal and family lives; and provide nurses with satisfying salary and benefits, high job security, and positive relationships with other nurses and with management. The other policies may appeal to groups of nurses, but they are not directed towards any of the major job satisfaction indicators.

Every year, the nurse attends a nursing conference and takes several continuing education courses to help maintain licensure. Which section of the ICN Code of Ethics does this uphold? A) Nurses and people B) Nurses and practice C) Nurses and the profession D) Nurses and co-workers

Answer: B Explanation: A) The nurses and practice section of the ICN Code of Ethics states that nurses carry the professional responsibility and accountability for nursing practice and for maintaining competence by continual learning. The other sections of the ICN Code of Ethics do not address continuing education for nurses.

7) The nurse is caring for a group of clients on a psychiatric unit. One client has become highly agitated and is threatening other clients and some of the staff. The nurse escorts the client to the isolation room, leaving the door open. Which statement by the nurse is most appropriate? A) "I can't believe you behaved in this manner and upset everyone else." B) "I'm going to stay here with you in case you want to talk about what happened." C) "If you do not calm down, I will sedate you for your own safety." D) "You scared the other clients."

Answer: B Explanation: A) The nurse offers support to the client by offering to listen if the client wants to talk about the episode. Telling the client that the nurse can't believe the behavior is not effective communication at this point. Threatening the client with medication is not allowed. The psychiatric client may not be aware enough to realize the impact of behavior on the other clients.

The nurse administers morphine to a client after surgery to help manage pain even though morphine has a risk of creating dependence and addiction. What ethical principle does the nurse apply in this situation when planning care? A) Veracity B) Justice C) Autonomy D) Beneficence

Answer: D Explanation: A) Beneficence requires that the actions one takes should promote good. This includes giving treatments that have some risks when the nurse and others involved in client care have determined that the benefits outweigh the risks. Autonomy is the right to self-determination. Justice means treating all clients fairly. Veracity is the principle of always telling the truth.

The nurse is concerned about the risk involved when implementing healthcare provider prescriptions for a newly admitted client. Which strategies should the nurse consider to reduce this risk? Select all that apply. A) Question any order written for a postoperative client. B) Question any order a client questions. C) Question any order if the client's condition changes. D) Question any verbal order. E) Question any order that is incomplete.

BCE

You are caring for a patient who will undergo a bone marrow aspiration, a difficult and painful procedure necessary to monitor the progress of recuperation after bone marrow transplantation. You are eager to minimize pain for this patient. You review the medical record for previous successful pain-management plans. You discuss the procedure with the patient. You advocate for the patient when the health care provider arrives to prepare for the procedure. Which ethical principle best describes the reasons for your actions? A. Beneficence B. Accountability C. Nonmaleficence D. Respect for autonomy

C Although all these principles are important and valuable for nursing practice, the principle of nonmaleficence best describes efforts to minimize pain, particularly when at least some pain is unavoidable. The term is Latin and translates literally as "non" indicating "not" and "maleficence" indicating "harmful act."

During an assessment of a child in the urgent care clinic, the nurse notes that the child has a swollen and split lip. When asking the parent how the child's lip injury occurred, the parent responds, "We are here for my child's ear not my child's lip." Which is the rationale for reporting this incident? A) The child reports that a parent caused the injury. B) The lip injury is unrelated to the ear infection. C) The nurse can be sued if there is no abuse. D) Suspected abuse must be reported.

D

d

Which nursing activity is a common cause of pediatric medical​ errors? a Communication with children b Monitoring adverse events c Administration of drugs in suspension d Mathematical calculation

A healthcare provider submits a reimbursement claim to Medicare for services that have not been rendered. Why is this action unethical and what is a potential consequence? a. It is fraudulent, and the healthcare provider may be fined. b. It is a conflict of interest, and the provider may have to repay the payment. c. It violates the Debt Collector Act, and the provider's license may be revoked. d. It is against the Fair Trade Act, and the healthcare provider may face legal action.

a. It is fraudulent, and the healthcare provider may be fined.

A 17-year-old has been diagnosed with a genetic disorder that results in 50 percent of the teenager's offspring carrying that gene. The teenager becomes pregnant and the teenager's mother demands the daughter have an abortion, but the teenager refuses. Which right is the teenager exercising? a. Fidelity b. Veracity c. Autonomy d. Beneficence

c. Autonomy

Genetic testing of a child reveals that the husband is not the child's biological father. The mother does not want the husband to know. Which right of the mother applies to her decision? a. Justice b. Veracity c. Confidentiality d. Informed consent

c. Confidentiality

An 86-year-old patient is suffering from a tumor and only has Medicare for insurance. The tumor will eventually grow to occlude the airway, so surgery is required even though the patient's medical history makes this a high-risk procedure. The patient must decide whether to proceed with the surgery. Which cost must this patient consider? a. Hospice care b. Probate court c. Hospitalization d. Affordable healthcare plan

c. Hospitalization

When the nurse attempts to administer a medication to a patient, the patient refuses to take the medication because it causes diarrhea. The nurse provides teaching about the medication, but the patient continues to adamantly refuse the medication. What should the nurse do first? 1. Document the patient's refusal to take the medication 2. Notify the practitioner of the patient's refusal to take the medication 3. Discuss with a family member the need for the patient to take the medication 4. Explain again to the patient the consequences of refusing to take the medication

1. CORRECT: Withholding the medication and documenting the patient's refusal are the appropriate interventions. Patient's have a right to refuse care. 2. Notifying the practitioner eventually should be done, but it is not the priority at this time. 3. Discussing the situation with a family member without the patient's consent is a violation of confidentiality. 4. The patient has been taught about the medication and adamantly refuses the medication. Further teaching at this time may be viewed by the patient as badgering.

When attempting to administer a 10:00 PM sleeping medication, the nurse assesses that the patient appears to be asleep. What should the nurse do? 1. Withhold the drug 2. Notify the practitioner 3. Awaken the patient to administer the drug 4. Administer it later if the patient awakens during the night

1. This is a violation of the practitioner's order. Drug administration is a dependent nursing function. 2. This is unnecessary. 3. CORRECT: Administering a medication is a dependent function of the nurse. The prescription should be followed as written if the prescription is reasonable and prudent. This medication was not a PRN medication but rather a standing order. 4. The drug should be administered as prescribed not at a later time.

c (Rationale A just culture environment considers healthcare errors the likely result of system​ failures, not natural individual mistakes. A just culture environment believes that an atmosphere of punishment can impede error prevention activities. A just culture environment will not tolerate employee gross misconduct. Just culture employees consider themselves stakeholders.)

A nurse is returning to work as an intensive care staff nurse after an absence of several years and is taking a continuing education course on just culture. What statement by the nurse shows the need for further education on this​ topic? ​a "Just culture employees consider themselves​ stakeholders." ​b "An atmosphere of punishment can impede error prevention​ activities." ​c "Healthcare errors are the likely result of natural individual​ mistakes." ​d "A just culture environment will not tolerate employee gross​ misconduct."

The nurse is planning to carry out advocacy interventions when caring for a client with brain cancer. Which value should the nurse recognize as most basic to client advocacy? A) The client is a holistic, autonomous being who has the right to make choices and decisions. B) The nurse has the responsibility to ensure the client's decisions guide care regardless of whether the client is mentally competent. C) Clients should be advised that making their own care decisions is almost invariably detrimental to their well-being. D) The client is a dependent being who has the right to expect the nurse to solve all healthcare needs.

Answer: A Explanation: A) Safeguarding clients' autonomy is the first core attribute of advocacy. It requires respecting and promoting each client's right to self-determination, except in those situations when the client is incompetent to decide or does not wish to be involved in decision making. Clients should not be discouraged from making their own decisions or be treated as naturally dependent.

Staff at the hospital have decided to strike in order to try to improve working conditions. Which are some of the ethical issues that a nurse working in the hospital will have to consider when deciding whether or not to honor the picket line? Select all that apply. A) The need to support coworkers in their efforts to improve working conditions B) The need to ensure that clients receive care and are not abandoned C) The desire to take some time off D) Loyalty to the nurse's employer E) The need for higher pay

Answer: A, B, D Explanation: A) Strikers may be concerned about client care as it is related to adequate staffing. Strikes may adversely affect client care and outcomes. Nurses may feel allegiance to a hospital where they have worked for years. The desire to take time off and the need for higher pay are not ethical issues.

A patient's visitor has fallen in the patient's room. Which of the following is the most appropriate action for the nurse to take? A. Call the nursing supervisor. B. Assist the visitor and document with an occurrence report. C. Assist the visitor and, if there is no injury, document nothing. D. Assist the visitor and document the occurrence in the patient's chart.

B The nurse's first action would be to assist the visitor to make sure that he or she is safe and then complete an occurrence report. An occurrence report is completed for any unanticipated event, whether it occurs to a patient or visitor. It is completed even though there is no apparent injury. An occurrence report is never referred to in the patient's chart. Calling the nursing supervisor is a proper action but only after the visitor has been made safe and the occurrence documented in an appropriate form.

A client who has stage IV pancreatic cancer decides to discontinue all treatment .Which of the following actions should the nurse take? A) Offer alternative medication. B) Encouraging the client to reconcile. C) Ask the client to discuss the decision. D) Request a mental health consultation.

C.

The nurse is interviewing a patient who has a persistent cough. The patient reports doing online research and using essential oils to treat the cough. Which ethical dilemma does this pose for the nurse? Deciding whether the patient should be told that the treatment is not effective Deciding whether to contact the manufacturer of the essential oils Deciding whether to continue the interview, because the patient is already self-treating Deciding whether to document that the patient is using essential oils to treat the cough

Deciding whether the patient should be told that the treatment is not effective

The nurse is performing care interventions for patients who lack moral development or have been exposed to corrupt environments. Which is an important self-care consideration for the nurse? Refraining from speaking to these patient unless necessary because they pose a safety risk Taking the opportunity to discuss their own moral code with patients in an effort to foster moral development Ensuring personal safety by taking such measures as getting between a patient and the door Instructing these patients about how they must be compliant in all aspects of care because the care team has formed a plan that implements morals and ethics

Ensuring personal safety by taking such measures as getting between a patient and the door

A child is taught early in life that it is wrong to take things from others. This process carries over to behavior later in life. Which process does this exemplify? Cultural relativism Moral development Utilitarianism Responsibilities

Moral development

The nurse administers blood to a patient without verifying the patient's identity. As a result, the patient receives the wrong type of blood and has a severe reaction. Which principle of ethical decision making is demonstrated by the nurse's failure to verify the correct blood? Veracity Justice Beneficence Nonmaleficence

Nonmaleficence

A highly skilled bariatric surgeon performs about one surgery per month at the facility, and the hospital is required to carry expensive insurance to cover bariatric procedures. The hospital decides to discontinue the service as a cost-saving measure. Which consequent-based theory is applied in this situation? Responsibility Utilitarianism Principles-based theory Relationship-based theory

Utilitarianism

A healthcare worker returns from an 11-week approved absence taken to care for a sick parent. The department manager demotes the employee for being absent too long. Which regulation did the department manager violate? a. Fair Labor Standards Act b. Family and Medical Leave Act c. Equal Employment Opportunity Act d. Workers' Compensation Disability Act

b. Family and Medical Leave Act

At a busy emergency room in the U.S., a doctor does not want to treat a patient because of the patient's foreign status. Which step should this doctor take? a. Activate the waiver of treatment protocol b. Treat the patient regardless of national origin c. Refuse to treat the patient under the Emergency Medical Treatment and Active Labor Act (EMTALA) provision d. Transfer the patient to another hospital under Affordable Care Act (ACA) regulations

b. Treat the patient regardless of national origin

The nurse is reviewing the American Hospital Association's (AHA's) patient responsibilities with a 19-year-old patient at 34 weeks of gestation. As the nurse is speaking, the patient interrupts and says, "I get it. You're in charge, and I have to do what you tell me." Which response by the nurse is most appropriate? "Patient responsibilities are established to make sure that you know you can refuse care without suffering any consequences." "Patient responsibilities are designed to help you and your healthcare providers work together as a team." "Patient responsibilities are intended to help keep you and members of your healthcare team safe and comfortable." "Patient responsibilities are meant to help you understand that you are entitled to respectful, courteous care."

"Patient responsibilities are designed to help you and your healthcare providers work together as a team."

The nurse notices that there is broken equipment on the playground in a neighborhood where care is provided to low-income residents. The nurse tries unsuccessfully to get the owner to address this safety issue. Which additional action is most consistent with the role of the nurse as a public advocate? A) Write an article in the local newspaper to gain public attention. B) No action is required; this is a civil problem beyond the realm of the nurse. C) Call the police to report the owner's neglect. D) Tell the parents they should not pay their rent until the playground is fixed.

Answer: A Explanation: A) Prevention of injury to clients is very much a part of nursing. The nurse should initially contact the owner. Because this was ineffective, it is appropriate for the nurse to write an article or talk to an individual on the town council. The police are a protection against crime and are not likely to do anything about the situation. Telling the parents not to pay their rent until the playground is fixed is not appropriate. Civil problem or not, the nurse has an obligation to protect the neighborhood children from injury and should act on it.

What are some reasons the nurse might withhold food and fluids from a client? Select all that apply. A) A competent and informed client refuses them. B) A son decides that it is his father's time to die. C) It is determined to be more harmful to administer them than to withhold them. D) A schizophrenic client believes that they are being poisoned. E) The nurse thinks that the client is in too much pain.

Answer: A, C Explanation: A) The autonomy of a competent and informed client must be respected. Family members cannot overrule client choices. Forcing an individual with terminal illness to eat, or starting artificial nutrition, will often make the client feel bloated, feel nauseated, and/or develop diarrhea. Clients must be capable of making informed choices. The nurse's opinion about the client's pain status is not a justification for withholding nourishment.

A nurse is volunteering time in a local free clinic that provides care to the underinsured population. By volunteering time to work in the clinic, this nurse is demonstrating which professional value? A) Human dignity B) Social justice C) Integrity D) Autonomy

Answer: B Explanation: A) Social justice is upholding fairness on a social scale. This value is demonstrated in professional practice when the nurse works to ensure equal treatment under the law and equal access to quality healthcare. Human dignity is respect for the worth and uniqueness of individuals and populations. Autonomy is respecting the client's right to make decisions about their healthcare. Integrity is acting in accordance with an appropriate code of ethics and accepted standards of practice.

The nurse does not administer pain medication to a patient as ordered and the patient experiences a great deal of pain and delayed recovery. Which best describes the liability of the nurse to provide pain medications? A. The nurse should have known in advance that the action would lead to complications. B. The nurse was legally obliged and responsible for providing pain control. C. The nurse intended to cause harm to the patient. D. The nurse had a legally enforceable obligation to adhere to a standard of care.

Answer: B The state of being legally obliged and responsible for delivery of pain medication means that the nurse had liability in this instance. Malpractice occurred when the nurse did not perform the obligatory responsibility. The nurse did not necessarily have a legally enforceable duty to provide the pain medication, but the failure to do so should have been communicated with the provider. Knowing in advance describes foreseeability, which the nurse may or may not have had in this instance. There is no way to determine whether the nurse had foreseeability or intended to cause harm.

The nurse is about to administer a medication to a patient and the patient states, "I am allergic to that medication." Which should be the next action by the nurse? A. Not giving the medication and documenting that the patient refused it B. Verifying that the patient does not have an allergy and administering the medication C. Verifying the order and contacting the physician D. Explaining to the patient that the physician wanted them to have it

Answer: C The order and patient history should be validated and verified. The provider should be contacted whether or not there is a documented allergy to Keflex. The medication should not be given until the allergy is verified. If an allergy is not documented, this may be an oversight, and the allergy should be added to the record. The medication should not be given, but documentation should not reflect that the patient refused it. The provider should be contacted and the order changed.

A patient who feels that they have been treated badly during an emergency department visit asks the nurse where to find relevant information to report this. Which is the appropriate response by the nurse? Asking the patient whether there is any specific issue they'd like to discuss and providing the information for reporting a complaint in the Patient's Bill of Rights Telling the patient that the information is available online but that the hospital will not provide it to them Asking the patient to please not report the facility or they may not be able to return for care Asking the patient whether they are planning to file a lawsuit and notifying risk management

Asking the patient whether there is any specific issue they'd like to discuss and providing the information for reporting a complaint in the Patient's Bill of Rights

Which of the following situations fails to meet the criteria for establishing nursing negligence or malpractice? A. A nurse comes to work under the influence of alcohol. B. The nurse leaves a client's bed in the raised position, resulting in a fall. C. The nurse fails repeatedly to document a client's response to pain medication. D. A nurse assigns first-time ambulation of a postop client to an aide and the client falls.

C The criteria require that the nurse-client contract be broken such as the alcohol scenario or preventable injury to the client as a result of the failure to follow good nursing practice. While failing to document is not acceptable practice, it would not be considered either malpractice or negligence unless the omission resulted in patient injury.

A 23-year-old Christian scientist has been hit by a car and taken to the emergency room (ER). The patient presents to the ER and complains of pain. The patient's leg appears to be broken, so the ER staff prepares to take the patient's X-rays. When the staff arrives, the patient becomes angry, refuses the X-rays, and leaves the emergency room. Which ethical factor influences this patient's refusal of treatment? a. Justice b. Religion c. Beneficence d. Nonmaleficence

b. Religion

A patient with hypertension goes to a clinic for a follow-up visit. Upon taking the patient's vital signs, the nurse notices the patient's blood pressure is far above normal. The patient states, "The cost of medication is unaffordable." How should the nurse respond? a. Call the pharmacist to seek a discount on the medication. b. Tell the patient to get free medication at the emergency room. c. Contact social services to assist with obtaining the medication. d. Ask the physician for a free continuing supply of the medication.

c. Contact social services to assist with obtaining the medication.

A person is found unconscious and taken to the emergency room. It is determined that the patient is bleeding from the head and requires emergency surgery. Surgery is immediately performed and the patient recovers. Which assumption did the healthcare providers act upon? a. Adequacy b. Autonomy c. Implied consent d. Power of attorney

c. Implied consent

The nurse administers an incorrect medication to a client and realizes the error a few minutes later. Which action should the nurse​ perform? A. Document the error but do not complete an incident report unless the client has an adverse reaction. B. Do not complete an incident report because the client did not experience any adverse effect. C. Complete an incident report and document the report in the medical record. D. Complete an incident report but do not document the report in the medical record.

d

The client who requires a co-signature for a valid consent for surgery is a: 1. 15-year-old mother whose infant requires exploratory surgery 2. 40-year-old resident in a home for developmentally disabled adults 3. 90-year-old adult who wants more information about the risks of surgery 4. 50-year-old unconscious trauma victim who needs insertion of a chest tube

1. A mother may legally make medical decisions for her children even if the mother is younger than 18 years of age. 2. CORRECT: A client living in a protected environment such as a home for developmentally disabled adults may not have the mental capacity to make medical decisions and requires the signature of a court-appointed legal representative. This person could be a parent, sibling, relative, or unrelated individual. 3. Older adults can make decisions for themselves as long as they understand the risks and benefits of the surgery and are not receiving medication that may interfere with cognitive ability. 4. The insertion of a chest tube to inflate a lung is an emergency intervention to facilitate respiration and oxygenation. This emergency procedure is implemented to sustain life and does not require a signed consent if the client is incapacitated.

The nurse is informed that a credentialing team has arrived and is in the process of assessing quality of care delivered at the hospital. What is the organization associated with the credentialing of hospitals? 1. Joint Commission 2. National League for Nursing 3. American Nurses Association 4. National Council Licensure Examination

1. CORRECT: The Joint Commission (formerly the Joint Commission on Accreditation of Healthcare Organizations) evaluates health-care organizations' compliance with Joint Commission standards. Accreditation indicates that the organization has the capabilities to provide quality care. In addition, federal and state regulatory agencies and insurance companies require Joint Commission accreditation. 2. The National League for Nursing (NLN) fosters the development and improvement of nursing education and nursing service. 3. The American Nurses Association (ANA) is the national professional organization for nursing in the United States. Its purposes are to promote high standards of nursing practice and to support the educational and professional advancement of nurses. 4. In the United States, graduates of educational programs that prepare students to become Licensed Practical Nurses or Registered Professional Nurses must successfully complete the National Council Licensure Examination-PN (NCLEX-PN) and the National Council Licensure Examination- RN (NCLEX-RN), respectively, as part of the criteria for licensure.

A practitioner asks the nurse to witness an informed consent. Which patient does the nurse identify is unable to give an informed consent for surgery? 1. 16-year-old boy who is married 2. 35-year-old woman who is depressed 3. 50-year-old woman who does not speak English 4. 65-year-old man who has received a narcotic for pain

1. Legally, individuals younger than 18 years old can provide informed consent if they are married, pregnant, parents, members of the military, or emancipated. 2. A depressed person is capable of making health-care decisions until proven to be mentally incompetent. 3. This person can provide informed consent after interventions ensure that the person understands the facts and risks concerning the treatment. 4. CORRECT: Narcotics depress the central nervous system, including decision-making abilities. This person is considered functionally incompetent.

The nurse completes an Incident Report after a patient falls while getting out of bed unassisted. What is the main purpose of this report? 1. Ensure that all parties have an opportunity to document what happened 2. Help establish who is responsible for the incident 3. Make data available for quality-control analysis 4. Document the incident on the patient's chart

1. The nurse who identified or created the potential or actual harm completes the Incident Report. The report identifies the people involved in the incident, describes the incident, and records the date, time, location, actions taken, and other relevant information. 2. Documentation should be as factual as possible and avoid accusations. Questions of liability are the responsibility of the courts. 3. CORRECT: Incident Reports help to identify patterns of risk so that corrective action plans can take place. 4. The report is not part of the patient's medical record, and reference to the report should not be made in the patient's medical record.

8. Which identifies accurate nursing documentation notations? Select all that apply a. The client slept through the night b. Abdominal wound dressing is dry and intact without drainage c. The client seemed angry when awakened for vital sign measurement d. The client appears to become anxious when it is time for respiratory treatments e. The client's left lower medial leg wound is 3 cm in length without redness, drainage, or edema

8. A, B, E- Factual documentation contains descriptive, objective information about what the nurse sees, hears, feels, or smells. The use of inferences without supporting factual data is not acceptable because it can be misunderstood. The use of vague terms, such as seemed or appears is not acceptable because these words suggest that the nurse is stating an opinion.

A nurse working on a medical-surgical unit wants to ensure care is provided within the standard of nursing care. Which actions by the nurse are appropriate? Select all that apply. A) Analyze the position description. B) Review and become familiar with the policy and procedure manual. C) Question the value of collaborating with other disciplines. D) Review applicable state nurse practice act and administrative rules. E) Adhere to national standards of practice and care.

A B D E

The nurse is caring for a client on a medical-surgical unit. The client tells the nurse that the healthcare provider has refused to treat the client further if the client continues to be noncompliant with the healthcare provider's recommendations. Which is the priority nursing action in this situation? A) Take the issue to the hospital ethics committee. B) Advise the client to sue the healthcare provider. C) Have the client contact a consumer agency. D) Notify the healthcare provider of the client's complaints.

Answer: A Explanation: A) Acting as a client advocate and protecting the client's rights, the nurse should enlist the help of the hospital ethics committee. The nurse never advises a client to sue but assists the client to find help resolving the issue. A consumer agency is not appropriate because this is an ethical matter. The nurse should act on behalf of the client, and the best way to do that is by taking the issue to the hospital ethics committee, not to the healthcare provider.

A nurse educator is talking to a student about how to deal with an ethical dilemma in practice. Which does the nurse educator explain to the student as important regarding actions during an ethical dilemma? A) Examining all conflicts in the situation B) Investigating all aspects of the situation C) Relying on nursing judgment D) Making a decision based on the policy of the agency

Answer: B Explanation: A) To avoid making a premature decision, the nurse plans to investigate all aspects of the dilemma before deciding. Overconfidence can lead to poor decision making. Reading the agency policy regarding the matter addresses only one aspect of the situation. Examining the conflicts surrounding the issue is only one aspect of the situation to consider.

A child is diagnosed with a fractured cheekbone and multiple bruises and the nurse suspects abuse. Which action should the nurse take? A. Calling the police immediately and having the parents arrested B. Not reporting the injury since it was reported as having been caused by a fall C. Documenting the suspicion and continuing with care D. Notifying the supervisor and contacting the appropriate local authorities

Answer: D Under good faith immunity, suspected abuse must be reported. Nurses should refer to their local facility policies related to the procedure for reporting. If abuse is suspected, a report must be made, whether the initial complaint is related to abuse or not. The nurse cannot facilitate an investigation or arrest related to the suspicion but should report as required. Documentation is important, but this is not the only thing the nurse should do.

A 9-year-old patient undergoing chemotherapy for leukemia tells the nurse that they have decided to refuse any future chemotherapy treatments because they make him so sick. Which is the most ethical action for the nurse to take in response to this statement? Telling the child that it is not their decision because they are a minor and do not have the right to refuse care Telling the child that they are making a very bad decision and that refusing chemotherapy will likely result in death Advising the parents to cancel future chemotherapy sessions because the child will refuse the treatment. Approaching the parents and telling them that the child is worried about future chemotherapy treatments, then helping facilitate a family discussion

Approaching the parents and telling them that the child is worried about future chemotherapy treatments, then helping facilitate a family discussion

Which statement should the nurse recognize as the primary role of risk management in a healthcare setting? A. Providing service recovery to prevent those who have suffered an injury from filing suit B. Reducing the possibility of patient injury and loss to the organization C. Training staff adequately to perform their jobs without risk of committing errors D. Gathering evidence to defend a hospital in the event of a lawsuit

B Risk management exists to decrease the likelihood of injury to a patient and to reduce the possibility of loss to the organization. Risk management may meet with those who have been injured in a healthcare facility and attempt to provide service recovery, but this is a strategy, not a primary function. Risk management is not responsible for training staff, but may make recommendations to improve the quality of care. Risk management does not directly work to defend the hospital in the event of a lawsuit but may identify elements of a situation that increased risk of injury.

The nurse is concerned about the risk involved when implementing healthcare provider prescriptions for a newly admitted client. Which strategies should the nurse consider to reduce this risk? Select all that apply. A) Question any order written for a postoperative client. B) Question any order a client questions. C) Question any order if the client's condition changes. D) Question any verbal order. E) Question any order that is incomplete.

B C E

Which program was specifically designed to increase patient engagement and set forth patient rights? The Joint Commission's Speak Up Campaign The Patient Self-Determination Act American Hospital Association Bill of Rights American Nurses' Association Code of Ethics

The Joint Commission's Speak Up Campaign

a,b,d ( Rationale The medications that increase the risk of making a pediatric medication error are those produced in adult​ concentrations, those needing a small dosage for an​ infant, and those calculated based on a​ child's weight. The risk is not as great for tablets needing to be crushed or liquids measured with an eye dropper dispenser.)

The pediatric oncology nurse is assigned to give medications to several children. For which medications would the nurse face an increased risk of making a medication​ error? ​(Select all that​ apply.) a Medication produced in adult concentrations b Medication calculated based on​ child's weight c Medication measured with an eye dropper dispenser d Medication needed in small dosage for an infant e Medication in the form of a tablet to be crushed

a,b (Two facts would disqualify nurses for protection under the Whistleblower Protection​ Act: the employer did not know that the activity was a violation of​ law, or the nurse made a​ verbal, not​ written, complaint to administration. To qualify for​ protection, the nurse must follow the​ employer's internal reporting​ procedures, and give the employer time to correct the issue. Also to qualify for​ protection, the employer has to threaten or engage in retaliation against the nurse.)

What facts would disqualify nurses for protection when reporting observed misconduct under the Whistleblower Protection​ Act? ​(Select all that​ apply.) a Nurse making a verbal complaint to administration b Employer not knowing that the activity was a violation of law c Employer engaging in retaliation against the nurse d Nurse following​ employer's internal reporting procedures e Nurse giving employer time to correct the issue

What is the primary purpose of the American Nurses Association Standards of Clinical Nursing Practice? 1. Establish criteria for quality practice 2. Define the philosophy of nursing practice 3. Identify the legal definition of nursing practice 4. Determine educational standards for nursing practice

1. CORRECT: The ANA Standards of Clinical Nursing Practice describe the nature and scope of nursing practice and the responsibilities for which nurses are accountable. 2. A philosophy incorporates the values and beliefs about the phenomena of concern to a discipline. The ANA Standards of Clinical Nursing Practice reflect, not define, a philosophy of nursing. Each nurse and nursing organization should define its own philosophy of nursing. 3. The laws of each state define the practice of nursing within the state. 4. Educational standards are established by accrediting bodies, such as the National League for Nursing Accrediting Commission, the Commission on Collegiate Nursing Education, and state education departments.

A practitioner writes a prescription for a medication that is larger than the standard dose. What should the nurse do? 1. Inform the supervisor 2. Give the drug as prescribed 3. Give the average dose of the medication 4. Discuss the prescription with the practitioner

1. It is unnecessary to call the supervisor in this situation. 2. This is unsafe for the patient and may result in malpractice. 3. Changing a medication prescription is not within the scope of nursing practice. 4. CORRECT: Nurses have a professional responsibility to know or investigate the standard dose for medications being administered. In addition, nurses are responsible for their own actions regardless of whether there is a written prescription. The nurse has a responsibility to question and/or refuse to administer a prescription that appears unreasonable.

During an interview, a graduate nurse asks the manager of the unit if the hospital participates in nursing research. The graduate tells the manager that participation in nursing research will most likely help the student: 1.Develop a better attitude about work. 2.Improve daily practice. 3.Improve the development of skills. 4.Improve communication with clients.

2

Based on current trends, which nursing care environment has the greatest employment potential for new nurses? A. Long-term care facilities B. Hospital emergency departments C. Pre- and postsurgical departments D. Primary health care provider practices

A Employment will not grow at the same rate in every setting—hospital employment will grow more slowly because many procedures and care are shifting to outpatient and home health settings, more sophisticated procedures can safely be done outside the hospital, and home health and long-term care facilities will see employment growth due to the aging population. The other options are more traditional environments and so are not as likely to show increased employment growth.

Which statement describes the application of utilitarianism? A hospital expands its emergency department services in order to provide more efficient care. A behavioral health agency closes three small rural clinics to expand and improve services provided in their busiest facilities. A nurse leaves a job in home hospice because of morality issues tied to providing pain medications. A physician practice adds a second provider to help increase the number of patients in the practice.

A behavioral health agency closes three small rural clinics to expand and improve services provided in their busiest facilities

d (To qualify for protection under the Whistleblower Protection​ Act, the nurse has to face a threat of or actual signs of retaliation from the employer. The nurse does not need to make appointments to further complain. The nurse does not need to request an OSHA inspection of the clinic. The nurse does not have to leave​ work, and resign the clinic position immediately.)

A clinic nurse discovers that all the fire extinguishers on the top two floors of the clinic are way past their expiration date. The nurse writes a letter to the​ clinic's Risk Manager and to the Safety Officer. After waiting a​ month, and receiving no reply from either​ individual, the nurse wrote again. This time the nurse added a statement about making a report to a state agency. What additional event must happen to have the nurse qualify for protection under the Whistleblower Protection​ Act? a The nurse must make appointments with the Risk Manager and the Safety Officer. b The nurse must leave​ work, and resign the clinic position immediately. c The nurse must request an OSHA inspection of the clinic. d The employer must threaten or engage in retaliation against the nurse for complaining.

Which of the following actions is/are required of the nurse practicing advocacy? (Select all that apply.) A. Speak up for patient care issues even when others may disagree. B. Contribute money toward the patient's health care costs if the patient is indigent. C. Assess the patient's point of view and prepare to articulate it. D. Document all clinical changes in the medical record in a timely and legible way. E. Become an active member of professional nursing organizations.

A, C, D Advocacy involves speaking up for the patient from the nurse's point of view, even when others may disagree and the advocacy is awkward or uncomfortable. To advocate well, the nurse needs to be sensitive to the patient's point of view by listening well and articulating accurately. Documentation about acts of advocacy helps others on the health care provider team to support your advocacy. Although providing charitable contributions can be praiseworthy, that act is not necessarily a nursing act based on nursing knowledge.

A nurse questions a medication prescription as too extreme in light of the client's advanced age and unstable statue. The nurse understands that this action is an example of which of the following ethical principles? A. fidelity B. autonomy C. justice D. nonmaleficence

A. INCORRECT: Fidelity is an agreement to keep promises. The nurse is not addressing a specific promise when she determines the appropriateness of a prescription for the client. Thus, this principle does not apply. B. INCORRECT: Autonomy is the right to make personal decisions, even when they are not necessarily in the person's best interest. No personal decision is involved when the nurse questions the client's prescription. C. INCORRECT: Justice is fairness in care delivery and in the use of resources. In this situation, the nurse is delivering responsible client care and not assessing available resources. This principle does not apply. D. CORRECT: Nonmaleficence is the avoidance of harm or injury. In this situation, administering the medication could harm the client. By questioning it, the nurse is demonstrating this ethical principle.

A mentally competent client with end-stage liver disease continues to consume alcohol after being informed of the consequences of this action. Which of the following best illustrates the nurses role as a client advocate? A. Asking the spouse to take all alcohol out of the house B. Accepting the client's choice and not intervening C. Reminding the client that the decision may be an end-of-life decision D. Refusing to care for the client due to the client's noncompliance

B.

The nurse is assisting with the informed consent process for a patient who is to undergo an elective procedure. The surgeon has spoken to the patient, and the nurse brings the consent form for the patient to sign. The patient states, "I've changed my mind. I don't want to do this." Which is the most appropriate action for the nurse to take? Informing the patient that they have the right to refuse the procedure and discharging the patient home after documenting the refusal Telling the patient that they do have the right to refuse the procedure, then notifying the healthcare provider that the patient is refusing Telling the patient that they should have told the healthcare provider before the consent form was prepared Informing the patient that the healthcare provider has already obtained verbal consent and that they must now sign the form

Telling the patient that they do have the right to refuse the procedure, then notifying the healthcare provider that the patient is refusing

An 18-year-old college student presents to the student health center after suspecting she is pregnant from nonconsensual sex. The teen's cultural beliefs are against abortion. Which approach is being used? a. Pluralist b. Utilitarian c. Duty-oriented d. Contractarian

b. Utilitarian

While making rounds on the night shift, a nursing supervisor notes that a 73-year-old client under observation following a myocardial infarction has multiple visible bruises on the arms and legs. The supervisor suspects abuse because nothing in the client's chart suggests this client should have sustained these injuries. This state's good faith immunity applies in cases of suspected abuse not only of children but also of older adults or adults with disabilities. Which action has the highest priority for the nursing supervisor in this situation? A) Notify authorities regarding the suspected abuse. B) Do nothing about the situation. C) Notify the security department. D) Ask a shift nurse about the source of the injuries.

A

5) The nurse is working in a low-income neighborhood as a home health nurse. During one of the home care visits to the neighborhood, the nurse notes that the children's playground has broken equipment, which the children continue to play on. The nurse has already contacted the owner of the playground about the issue. What action might the nurse take as a public advocate? A) Write an article to the local newspaper to gain public attention. B) No action is required; this is a civil problem beyond the realm of the nurse. C) Call the police to report the owner's neglect. D) Tell the parents they should not pay their rent until the playground is fixed.

Answer: A Explanation: A) Prevention of injury to client is very much a part of nursing. The nurse should initially contact the owner. Because this was ineffective it is appropriate for the nurse to write an article or talk to an individual on the town council. The police are a protection against crime and are not likely to do anything about the situation. Telling the parents not to pay their rent until the playground is fixed is not appropriate. Civil problem or not, the nurse has an obligation to protect the neighborhood children from injury and should act on it.

Which action demonstrates correct reporting of suspected child abuse? A) The nurse includes the entirety of the client's medical record. B) The nurse compiles a report with all pertinent information that is factually true. C) The nurse recommends that the organization report the abuse to state authorities. D) The nurse reports only information the client has authorized for release.

B

A baby is born with a severe neural tube defect. Healthcare providers explain to the parents that a surgical intervention is required for the child to live, and the parents decline the surgery. Which right are the parents exercising? a. Autonomy b. Beneficence c. Informed consent d. Withdrawal of treatment

a. Autonomy

Which factor is unique to malpractice when comparing negligence and malpractice? 1. The action did not meet standards of care 2. The inappropriate care is an act of commission 3. There is harm to the patient as a result of the care 4. There is a contractual relationship between the nurse and patient

1. There is a violation of standards of care with both negligence and malpractice. 2. Negligence and malpractice both involve acts of either commission or omission. 3. The patient must have sustained injury, damage, or harm with both negligence and malpractice. 4. CORRECT: Only malpractice is misconduct performed in professional practice, where there is a contractual relationship between the patient and nurse, which results in harm to the patient.

10. Nursing staff members are sitting in the lounge taking their morning break. An unlicensed assistive personnel (UAP) tells the group that she thinks that the unit secretary has acquired immunodeficiency syndrome (AIDS) and proceeds to tell the nursing staff that the secretary probably contracted the disease from her husband, who is supposedly a drug addict. Which legal tort has the UAP violated? a. Libel b. Slander c. Assault d. Negligence

10. B- Defamation is a false communication or a careless disregard for the truth that causes damage to someone's reputation, either in writing (Libel) or verbally (slander). An assault occurs when a person puts another person in fear of a harmful or an offensive contact. Negligence involves the actions of professionals that fall below standard of care for a specific professional group

A student nurse administers a medication to the wrong client while the instructor is with another student. Which statement by the instructor is most appropriate in this situation? A) "You have placed the nursing student program in danger." B) "You may be sued by the hospital for the extra care cost to the client." C) "You are expected to practice like a licensed nurse." D) "You have set a bad example for the other students."

Answer: C Explanation: A) A nursing student is held to the standard of conduct of an experienced, licensed professional nurse. Students are required to know the standards and to follow them. Hospitals do not generally sue nurses to recover money for extended care due to an error. It is not likely that the teaching program is in danger, as people do make mistakes and hospitals do rely on nursing schools to help provide care to clients. It is not likely that the other students are apt to follow the example of a student who fails to follow policy.

An adolescent client with a sexually transmitted infection (STI) says to the nurse, "Promise you won't tell my parents about my condition." The agency policy is that all STIs must be reported in accordance with federal and state law. Which action by the nurse is appropriate? A) Disclosing information to the parents B) Reporting the STI to the proper authorities C) Respecting the client's privacy and confidentiality by not mentioning or reporting the STI D) Telling other nurses in the clinic that the client has an STI

B

An older adult patient is increasingly unable to care for himself. The patient has had several falls and his children are worried about his ability to live alone safely. Which is the most frequently encountered patient rights issue related to older adult patients? Deciding whether or not to have a do-not-resuscitate (DNR) order Designating of a power of attorney for healthcare Deciding on long-term care in nursing homes or skilled nursing facilities Getting providers to listen and take their complaints seriously

Deciding on long-term care in nursing homes or skilled nursing facilities

The nurse is obtaining a medical history from a patient who has come from a corrupt background. The nurse quickly identifies that the patient is lacking in moral development. Which is the best approach to assessing the patient? Refraining from asking any further questions as part of the assessment Maintaining a calm, nonjudgmental attitude and ask open-ended questions Documenting that the results of the interview are invalid because the patient cannot participate responsibly Taking opportunities to correct the patient when he says something that is incorrect

Maintaining a calm, nonjudgemental attitude and ask open-ended questions

The nurse is caring for an 66-year-old older woman who has identified her next-door neighbor as her advocate and support person. Which is the appropriate action by the nurse under The Joint Commission's Speak Up Campaign guidelines? Respecting the patient's wishes and allowing the neighbor to serve as the patient's advocate Informing the neighbor that the patient is likely confused and thinks she is a family member Informing the patient that the support person or advocate must be a relative Documenting that no next of kin is available

Respecting the patient's wishes and allowing the neighbor to serve as the patient's advocate

b,d,e (Rationale Some forms of job discrimination include not rehiring a nurse into a former​ position, reducing a​ nurse's salary, and decreasing a​ nurse's scheduled hours. Assigning a new graduate nurse to the night shift and putting a nurse on the weekend coverage schedule are not forms of job discrimination.)

Several nurses attend an educational lecture by an official from the Occupational Safety and Health Administration​ (OSHA) and are reminded about the Whistleblower Protection Program. They hear about the various forms that job discrimination could​ take, which is prohibited by law. What are some forms of job​ discrimination? ​(Select all that​ apply.) a Putting a nurse on the weekend coverage schedule b Reducing the salary that a nurse is paid c Assigning a new graduate nurse to the night shift d Decreasing scheduled hours that a nurse works e Not rehiring a nurse into a former position

A nurse was present when the doctor told a patient of the confirmed diagnosis of advanced cancer. Because the patient's spouse is a friend of the nurse, the nurse called the spouse to provide information about the diagnosis. Which professional behavior has been violated? a. HIPAA b. Civil rights c. Good judgment d. Patient advocacy

a. HIPAA

A hospital patient is placed on a ventilator after having a heart attack. The hospital staff believes the patient will never regain consciousness and will die in two to six months. The family is divided on whether or not to remove the patient from life support. What is the role of the healthcare ethics committee in advising on this case? a. To promote the doctor's clinical recommendations b. To promote shared decision making among stakeholders c. To determine priorities based on cost and resource allocation d. To determine whether state law or religious belief takes preference

b. To promote shared decision making among stakeholders

A physician working in a hospital refers a Medicare patient to a radiology clinic in town. The physician does not disclose that he owns the clinic. Why is this failure to disclose considered unethical? a. It is malpractice. b. It violates HIPAA. c. It is a conflict of interest. d. It violates the False Claims Act.

c. It is a conflict of interest.

A patient has been brought to an emergency department with severe blood loss and needs a transfusion immediately. However, the patient, who is awake and appears competent, has refused to accept the blood transfusion because of religious beliefs. What are the requirements for the hospital? a. The hospital must have chaplains on duty to address patient needs. b. The ethics committee must determine the hospital's best course of action. c. Under the Patient Self Determination Act, all providers must comply with the patient's request in order to receive reimbursement under Medicare. d. Under the Emergency Medical Treatment and Labor Act, the hospital must do everything it can to save the patient regardless of what the patient has stated.

c. Under the Patient Self Determination Act, all providers must comply with the patient's request in order to receive reimbursement under Medicare.

A 55-year-old patient has a rare autoimmune disorder with no cure. However, there are non-approved hospital-based treatments shown to be somewhat effective in controlling the symptoms and possibly prolonging life. The patient does not have the financial means to afford these uncovered treatments. What is a financial recourse for this patient? a. Finding available state funding b. Getting approval from Medicare c. Changing hospital insurance coverage d. Asking the hospital for a payment plan

d. Asking the hospital for a payment plan

In a hospital setting, a caregiver must inform a patient there is no cure for the patient's disease. To avoid possibly causing serious psychological harm by telling the patient, "There is nothing else I can do for you," the caregiver tells the patient, "Although there is currently no cure for your disease, I will treat your symptoms and make you more comfortable. I will also keep you informed as to any significant research regarding development of a cure for your disease." Which ethical principle is being used in resolving this dilemma? a. Justice b. Veracity c. Autonomy d. Nonmaleficence

d. Nonmaleficence

A comprehensive cancer center is seeking subjects for an experimental drug for prostate cancer. The drug is expected to have an 80% chance of improving the subject's condition but a 20% chance of either having no improvement or making it worse. Which aspect of informed consent should the subjects be ensured in this situation? a. They will be compensated appropriately. b. They will be required to complete the study. c. They will have their mental status assessed. d. They will understand the expected outcomes.

d. They will understand the expected outcomes

A nurse observes an assistive personnel (AP) reprimanding a client for not using the urinal properly. The AP tells him she will put a diaper on him if he does not use the urinal more carefully next time. Which of the following torts is the AP committing? A: Assault B: Battery C: False imprisonment D: Invasion of privacy

A: CORRECT: By threatening the client, the AP is committing assault. Her threats could make the client become fearful and apprehensive. B: INCORRECT: Battery is actual physical contact without the client's consent. Because the AP has only verbally threatened the client, battery has not occurred. C: INCORRECT: Unless the AP restrains the client, there is no false imprisonment involved. D: INCORRECT: Invasion of privacy most often involves disclosing information about a client to an unauthorized individual.

Which of the following is an advocacy intervention that a nurse may perform? A) Ensuring that clients and their families understand their legal rights. B) Deciding whether clients need to know information regarding their care. C) Following organizational policies and procedures in all cases without question. D) Leaving monitoring of clients' care to the clients themselves.

Answer: A Explanation: A) Educating clients and their families about their legal rights regarding informed decision-making is a specific advocacy intervention a nurse may make. Nurses should ensure that clients have all the information they need to give informed consent. They should review organizational policies and procedures to ensure protection of client rights, and they should monitor client care to ensure client rights.

9) What are some important nursing advocacy interventions? Select all that apply. A) Educating clients and their families about their legal rights B) Ensuring that clients have the necessary information to make an informed decision or give informed consent C) Speaking out for safe practice conditions when threatened by budget cutbacks D) Supporting medical authority E) Questioning other healthcare professionals when they provide care that is based on stereotypic ideas rather than on an assessment of the individual client's needs

Answer: A, B, C, E Explanation: A) Clients must understand their rights in order to be able to defend them. As an advocate, the nurse provides clients with the information they need to make informed decisions and supports the clients' rights to make their own healthcare decisions. A nurse should understand that advocacy may require political action. Conflicts may arise over issues that require consultation, confrontation, or negotiation between the nurse and administrative personnel or between the nurse and primary care providers. Nursing advocates should respect the diversity of all clients and promote equal care for all clients.

13) The nurse taking care of a Vietnamese client knows to respect the client's family's cultural views and honor traditions regarding healthcare decision making. In order to be an effective advocate, which factor or factors are important to the nurse? Select all that apply. A) Being assertive and recognizing the rights and values of clients and families B) Being aware that conflicts may arise C) Understanding that advocacy may require political action D) Evaluating advocacy decisions E) Honoring societal differences

Answer: A, B, C, E Explanation: A) It is important for the nurse to remember that client control over health decisions is a Western view and is not necessarily accepted in other cultures. The nurse must respect the client's and family's views and honor their traditions regarding healthcare decision making. To be an effective advocate for all people and cultures, the nurse must do the following: be assertive; recognize that the rights and values of clients and families must take precedence over those of healthcare providers; be aware that conflicts may arise over issues that require consultation, confrontation, or negotiation between the nurse and administration or the primary care provider; work with community agencies and lay practitioners; and understand that advocacy may require political action.

The nurse is in the midst of a complicated client care situation and is not sure what needs to be done with some information. Which healthcare issues must the nurse report to the state? Select all that apply. A) Amputation of a limb B) Death of a client C) Death of a neonate D) Diagnosis of tuberculosis E) Kidney transplant

Answer: B, C, D Explanation: A) The term mandatory reporting refers to a legal requirement to report an act, event, or situation that is designated by state or local law as a reportable event. All states mandate the reporting of certain vital statistics, including deaths. Many states also require healthcare providers to report neonatal deaths. Federal and state laws mandate the reporting of communicable diseases such as tuberculosis. Limb amputations and transplants do not need to comply with mandatory reportable events.

A client comes to the clinic and is found to have a sexually transmitted infection (STI). The client states to the nurse, "Promise you won't tell anyone about my condition." According to the Health Insurance Portability and Accountability Act (HIPAA) of 1996, which action must the nurse take? A) Honor the client's wishes. B) Respect the client's privacy and confidentiality. C) Communicate only necessary information. D) Not disclose any information to anyone.

Answer: C Explanation: A) HIPAA includes standards that protect the confidentiality, integrity, and availability of data as well as standards that define appropriate disclosures of identifiable health information and client rights protection. Nurses are entrusted with sensitive information, which at times must be revealed to other healthcare personnel in order to provide appropriate healthcare. In this case, the nurse may be required to report information to the state health department. Clients must be able to trust that their information is secure and will only be shared with appropriate entities. Nurses should not make promises to keep necessary information private.

The public's right to expect to receive treatment that meets the standards of nursing care is protected by: A. state law. B. federal law. C. reporting law. D. common law.

B Federal laws have a major effect on nursing practice, mandating a minimal standard of care in all health care settings that receive federal funds. Nursing practice is governed by state laws that delineate the conduct of licensed nurses and define behaviors of all health care professionals in promoting public health and welfare. Some states have enacted statutes that mandate nurses to report unsafe, illegal, or unethical practices of nursing colleagues or physicians. Common law is created through cases heard and decided in federal and state appellate courts. Throughout the years, judge-made law regarding nursing practice has accumulated in the form of written opinions.

The nurse planning to administer a blood pressure medication finds that the client is​ pale, diaphoretic, and tachycardic and has a blood pressure of​ 60/44 mmHg. Which action should the nurse​ take? A. Holding the medication and notifying the provider B. Giving the medication as ordered C. Giving the medication but continuing to monitor the client D. Repeating vital signs in 30 minutes

a

The practitioner orders OOB for a patient. How is the nurse functioning when moving this patient out of bed to a chair? 1. Dependently 2. Independently 3. Collaboratively 4. Interdependently

1. CORRECT: Determining the extent of activity desirable for a patient is within the practitioner's, not a nurse's, scope of practice. Following activity orders is a dependent function of the nurse. 2. The responsibility to determine a patient's activity level is not within the legal scope of nursing practice. 3. A practitioner works independently when determining a patient's desired activity level. 4. The nurse is following the practitioner's order to get the patient OOB. There are no restrictions or parameters in relation to the order. However, the nurse must use judgment before, during, and after a transfer if a patient's condition changes

d (Rationale The​ nurse's best approach is to contact the healthcare provider to clarify the stat nature of the lab order. The nurse should do that before waking up the​ client, talking to staff covering the next​ shift, or writing up an incident report.)

A night nurse makes two telephone calls to the covering healthcare provider during the early morning hours of the shift. Then the nurse discovers an order for a stat laboratory test that was​ requested, but not​ done, during daytime hours. Which action by the nurse is the most appropriate in this​ situation? a Write up an incident report about the error b Wake up the client for transport to the lab c Communicate the missed order to the next shift d Call the healthcare provider to clarify the need for the stat lab test

A client is about to undergo an elective surgical procedure. Which of the following actions are appropriate for the nurse who is providing preoperative care regarding informed consent? (Select all that apply.) A: Make sure the surgeon obtained the client's consent. B: Witness the client's signature on the consent form. C: Explain the risks and benefits of the procedure. D: Describe the consequences of choosing not to have the surgery. E. Tell the client about alternatives to having the surgery.

A: CORRECT: It is the nurse's responsibility to verify that the surgeon obtained the client's consent and that he understands the information the surgeon gave him. B: CORRECT: It is the nurse's responsibility to witness the client's signing of the consent form, and to verify that he is consenting voluntarily and appears to be competent to do so. The nurse also should verify that he understands the information the surgeon gave him. C: INCORRECT: It is the surgeon's responsibility, not the nurse's, to explain the risks and benefits of the procedure. D: INCORRECT: It is the surgeon's responsibility, not the nurse's, to describe the consequences of choosing not to have the surgery. E. INCORRECT: It is the surgeon's responsibility, not the nurse's, to tell the client about any available alternatives to having the surgery.

A patient is scheduled to have surgery, and informed consent is to be obtained. Place these steps in the order in which they should performed. 1. The patient is willing to sign the consent voluntarily 2. The patient signs the consent in the presence of the nurse 3. The nurse determines that the patient is alert and competent to give consent 4. The practitioner informs the patient of the risks and benefits of the procedure

Answer: 4, 3, 1, 2 4. It is the responsibility of the practitioner to provide all the information necessary to make a knowledgeable decision. Patients have a legal right to have adequate and accurate information to make informed decisions. 3. Patients must be competent to sign a consent form. The patient must be alert, competent, and in touch with reality. Confused, sedated, unconscious, or minor patients may not give consent. Minor patients who are married, parents, emancipated, or serving in the United States military can provide a legal consent. 1. Patients must give their consent voluntarily and without coercion. 2. The health-care provider witnessing the signing of the consent must ensure that the signature is genuine.

A hospice nurse is working with a client who has ovarian cancer. The client is concerned that her two daughters are at an increased risk for cancer and asks for the nurse's help. Which actions by the nurse are appropriate? Select all that apply. A) Provide the family with information on hereditary cancer risks. B) Assure the client that ovarian cancer is not hereditary. C) Offer to refer the daughters to a genetic counselor. D) Arrange for the client to have genetic testing. E) Tell the client that her additional worrying is too stressful.

Answer: A, C, D Explanation: A) A nurse's role as educator is crucial to ethical practice. Inaccurate reassurance or avoidance does not respect client rights. Providing appropriate alternatives and options for the client and the family are correct responses to the client's concerns.

The nurse is caring for a client on a mental health unit who is yelling at other clients and some of the staff. Which verbal intervention by the nurse is most consistent with the concept of advocacy? A) "You should be ashamed of your behavior. No wonder you ended up on a mental health unit." B) "You seem upset. Can you tell me what you think might help to calm you down?" C) "You need to behave. If this doesn't stop you are going to be placed in restraints." D) "You are out of control. You have no choice but to take more medication."

Answer: B Explanation: A) The nurse's role is to advocate for the rights of the individual with mental illness or disability. The nurse should validate the meaning of the behavior and encourage safe coping methods. Disparaging the client or threatening to restrain them or sedate them is inconsistent with client rights.

A 16-year-old girl reports being sexually active and taking birth control pills. The patient tells the nurse that her parents don't know this and would kick her out of the house if they did. Which action should the nurse take? Telling the patient not to worry, that this information will not be documented Informing the parents because the patient is a minor Respecting the right to confidentiality provided for adolescents Telling the patient that the parents wouldn't really kick them out

Respecting the right to confidentiality provided for adolescents

A doctor explained an urgent surgical procedure to a patient. The patient indicated she understood, agreed, and requested to talk to her husband prior to the surgery. Which action will demonstrate patient-centered care value? a. Allow the patient time to confer with her spouse b. Discourage the patient's request due to lack of time c. Offer to communicate with the husband on her behalf d. Ask the patient why she wants to speak with her husband

a. Allow the patient time to confer with her spouse

A female with a severe mental handicap has been brought to a clinic by the representatives of a state mental institution to undergo sterilization to avoid unwanted pregnancies. What allows the state to override an individual's rights? a. Eugenic sterilization b. Necessary sterilization c. Institutional sterilization d. Therapeutic sterilization

a. Eugenic sterilization

A vendor offers the logistics department manager at a community hospital an all-expenses-paid trip for four to a ski lodge. The manager went on the trip. Why did a potential conflict of interest occur? a. The manager had the opportunity to network during the trip. b. The manager used the relationship for personal financial gain. c. The manager was absent so department productivity would decline. d. The manager was rewarded for work completed by other staff.

b. The manager used the relationship for personal financial gain.

A pregnant woman has had a non-invasive genetic screening, which has indicated a chromosomal disorder in the fetus. The woman has signed consent for a first trimester abortion. The father opposes the abortion. What are the rights of the mother and father? a. The mother must involve the father in consenting for abortion b. The mother may consent for the abortion without the father's consent c. The father may cancel the procedure after talking with the obstetrician d. The father has the right to withhold consent for this on religious grounds

b. The mother may consent for the abortion without the father's consent

The nurse administers an incorrect dose of a medication to a patient. What is the primary purpose of documenting this event in an Incident Report? 1. Record the event for future litigation 2. Provide a basis for designing new policies 3. Prevent similar situations from happening again 4. Ensure accountability for the cause of the accident

1. Although documentation of an incident may be used in a court of law, it is not the primary reason for an Incident Report. 2. This is not the primary reason for Incident Reports. New policies may or may not have to be written and implemented. 3. CORRECT: Risk-management committees use statistical data about accidents and incidents to identify patterns of risk and prevent future accidents and incidents. 4. Although nurses are always accountable for their actions, accountability for the cause of an incidence is the role of the courts.

3. The nurse has just assisted a client back to bed after a fall. The nurse and health care provider have assessed the client and have determined that the client is not injured. After completing the incident report, the nurse should implement which action net? a. Reassess the client b. Conduct a staff meeting to describe the fall c. Document in the nurse's notes that an incident report was completed. d. Contact the nursing supervisor to update information regarding the fall

3. A- After a client's fall, the nurse must frequently reassess the client because potential complications do not always appear immediately after the fall. The client's fall should be treated as private information and shared on a "need to know" basis. Communication regarding the event should involve only the individuals participating in the client's care. An incident report is a problem-solving document; however, its completion is not documented in the nurse's notes. If the nursing supervisor has been made aware of the incident, the supervisor will contact the nurse if status update is necessary.

14) The home health nurse assessing a client notices the client's pain medication is missing 2 weeks' worth of medicine. The client says the only other individual besides she and the nurse who has been in her home is the other healthcare nurse. The nurse suspects her colleague took the medicine and must follow which guideline? A) Call the state board of nursing. B) Confront the nurse according to agency policy. C) Call the authorities immediately. D) Have the client file a police report.

Answer: A Explanation: A) Nurses have a legal responsibility to report any professional whom they suspect of engaging in illegal, immoral, or unethical activities. Normally, the nurse making such a report will do so following established procedures at the facility at which the nurse is employed. A nurse who suspects another nurse of impairment should also follow guidelines set forth by the board of nursing for the state in which he or she works. At other times, the nurse may need to seek the guidance of the state board of nursing or the American Nurses Association (ANA). A nurse who suspects a colleague of engaging in illegal, immoral, or unethical conduct and fails to act is in direct violation of the ANA Code of Ethics for Nurses (ANA, 2010).

6) The nurse has been working in a long-term care facility for 1 week. The nurse notes that during the evening meal, an unlicensed assistive personnel (UAP) takes a tray to a client who is unable to cut up and eat the food independently. The nurse helps the client with dinner and plans which of the following to advocate for the client? A) Report the UAP for neglect. B) Notify the physician. C) Call the client's family to have them assist with evening meals. D) Discuss the situation with the director of nursing.

Answer: D Explanation: A) The nurse would advocate getting the client's plan changed because the goal is to have someone available to help the client eat for every meal. Notifying the doctor will not help the client. The family might be able to help at times but cannot be expected to come for every meal. The nurse assesses that this happens at every meal and seeks to change how this client is cared for, not just changing one healthcare worker. The UAP is not neglecting the client. The UAP is assigned tasks by the nurse in charge of the client.

Ethics of care suggests that you resolve an ethical dilemma by attention to relationships. As Madeleine Leininger described it, caring is the "central and unifying domain for the body of knowledge and practices in nursing." How does it differ from other approaches to ethical dilemmas? (Select all that apply.) A. Ethics of care applies exclusively to nursing practice. B. Ethics of care pays special attention to the stories of the people involved in an ethical issue. C. Ethics of care uses logic and intellectual analysis based on universal philosophical principles. D. Ethics of care depends less on universal principles than other approaches to analyze ethical dilemmas. E. Stories about relationships can be distracting when trying to resolve an ethical dilemma.

B, C Ethics of care focuses on relationships, logic, and narrative as a way to understand the source of ethical dilemma and the resolution of dilemmas. It is not exclusive to nursing but applies to all areas of health care. It specifically depends more on storytelling and examination of relationships than on an analysis of universal principles.

A nurse believes that a pediatric patient has been the victim of abuse based on verbal statements and scarring noted on the patient's abdomen and legs. Which of the following is the best action for the nurse to take? A. Do nothing but document the patient's condition. B. Contact the patient's family. C. Contact the patient's teacher. D. Contact the Child Abuse Hotline.

D Nurses are mandatory reporters for suspected child abuse. Although documenting the patient's condition is important, it is insufficient to meet the mandatory reporting requirement. Contacting the patient's teacher and/or family does not meet the mandatory reporting requirement.

Once a month, the nurses working in case management get together for a potluck lunch. They have found that this informal session promotes a sense of a united community. Which of the International Council of Nurses Code of Ethics' relationships is enhanced by this practice? Nurses and coworkers Nurses and practice Nurses and people Nurses and the profession

Nurses and coworkers

A patient with advanced metastatic cancer is receiving frequent doses of pain medications and is unresponsive. The family agrees that life-sustaining measures be withdrawn. Pain medications are administered and the patient dies the next day. Which observation of the family indicates that the nursing interventions were successful and the family members are satisfied that the best decision was made? The family is talking easily about funeral arrangements and is being supported by a clergy member. The family is visibly grief-stricken and refusing to leave the room. The family asks the nurse whether they did the right thing. The family has become quiet and withdrawn and is having difficulty choosing a funeral home.

The family is talking easily about funeral arrangements and is being supported by a clergy member

A patient is hospitalized after a major traffic accident. The physician is paged to come to the hospital as soon as possible but decides to attend a previously scheduled meeting instead. By the time the physician reaches the hospital, the patient has lost consciousness due to severe blood loss. Which professional behavior did this physician ignore? a. First do no harm b. Patient self-autonomy c. Using knowledge correctly d. The most good for the most people

a. First do no harm

A patient participated in a clinical research trial that tests the effects of a new cancer-fighting drug. The patient develops a serious complication and dies. The patient's spouse, who was at every clinic visit, is convinced the patient was not provided alternatives to participation in the study. What is the alleged patient right violation? a. Authority b. Adequacy c. Autonomy d. Accountability

b. Adequacy

A teenager suffered massive head trauma in an automobile accident and has been in a prolonged persistent vegetative state. The teen's mother wants all treatment stopped but the teen's father wants everything done. Who has the authority to decide the rights of the patient in this situation? a. The mother can make the decision to discontinue life supporting care. b. The doctor can make the decision to continue or discontinue life supporting care. c. A court order can make the decision to continue or discontinue life supporting care. d. The ethics committee can make the decision to continue or discontinue life supporting care.

c. A court order can make the decision to continue or discontinue life supporting care.

A patient is going to have a surgical procedure to remove a brain tumor. The patient's neurosurgeon is part of a research study in which patients' tumors are collected and stored. What must the surgeon obtain to keep the patient's tumor for research? a. Patient waiver b. Tumor registry c. Informed consent d. Research authority

c. Informed consent

The Patient Protection and Affordable Care Act (PPACA) of 2010 is projected to expand health insurance coverage to an estimated 30 million people. How has this act expanded coverage? a. It has subsidized insurance companies. b. It has created a universal healthcare plan. c. It has allowed for a major Medicaid expansion. d. It has lowered the age limit for Medicare eligibility.

c. It has allowed for a major Medicaid expansion.

A patient is asked to participate in a medical research study. The nurse describes to the patient and family members how the patient is protected by the: 1. Code of Ethics 2. Informed Consent 3. Nurse Practice Act 4. Constitution of the United States

1. A code of ethics is the official statement of a group's ideals and values. It includes broad statements that provide a basis for professional actions. 2. CORRECT: Informed consent is an agreement by a client to accept a course of treatment or a procedure after receiving complete information necessary to make a knowledgeable decision. 3. Nurse Practice Acts define the scope of nursing practice; they are unrelated to informed consent. 4. The Constitution of the United States addresses broad individual rights and responsibilities. The rights related to nursing practice and patients include the rights of privacy, freedom of speech, and due process.

When considering legal issues the word contract is to liable as standard is to: 1. Rights 2. Negligence 3. Malpractice 4. Accountability

1. Although patients have a right to receive care that meets appropriate standards, the word right does not have the same relationship to the word standard as the relationship between the words contract and liable. 2. The words standards and negligence do not have the same relationship as contract and liable. Negligence involves an act of commission or omission that a reasonably prudent person would not do. 3. The words standards and malpractice do not have the same relationship as contract and liable. Malpractice is negligence by a professional person. 4. CORRECT: Liable means a person is responsible (accountable) for fulfilling a contract that is enforceable by law. Accountable means a person is responsible (liable) for meeting standards, which are expectations established for making judgments or comparisons.

The nurse must administer a medication. What should the nurse do first? 1. Check the patient's identification armband 2. Ensure the medication is in the medication cart 3. Verify the practitioner's prescription for accuracy 4. Determine the appropriateness of the prescribed medication

1. Although this action is essential for the safe administration of a medication to a patient, it is not the first step of this procedure. 2. Although this may be done as a timemanagement practice, it is not the first step when preparing to administer a medication to a patient. 3. CORRECT: The administration of medications is a dependent function of the nurse. The practitioner's prescription should be verified for accuracy. The prescription must include the name of the patient, the name of the drug, the size of the dose, the route of administration, the number of times per day to be administered, and any related parameters. 4. A nurse is legally responsible for the safe administration of medications; therefore, the nurse should assess if a medication prescription is reasonable. However, this is not the first step when preparing to administer a medication to a patient.

1. The nurse hears a client calling out for help, hurries down the hallway to the client's room, and finds the client lying on the floor. The nurse performs an assessment, assists the client back to bed, notifies the health care provider of the incident, and completes an incident report. Which statement should the nurse document on the incident report? a. The client fell out of bed b. The client climbed over the side rails c. The client was found lying on the floor d. The client became restless and tried to get out of bed.

1. C- The incident report should contain the client's name, age, and diagnosis. The report should contain a factual description of the incident, any injuries experienced by those involved, and the outcome of the situation. The correct option is the only one that describes the facts as observed by the nurse. Options 1, 2, and 4 are interpretations of the situation and are not factual information as observed by the nurse.

When caring for a terminally ill patient a family member says, "I need your help to hasten my mother's death so that she is no longer suffering." What should the nurse do based on the position of the American Nurses Association in relation to assisted suicide? 1. Not participate in active euthanasia 2. Participate based on personal values and beliefs 3. Participate when the patient is experiencing severe pain 4. Not participate unless two practitioners are consulted and the patient has had counseling

1. CORRECT: Nursing actions must comply with the law, and the law states that euthanasia is legally wrong. Euthanasia can lead to criminal charges of homicide or civil lawsuits for providing an unacceptable standard of care. 2. A nurse's beliefs, values, or moral convictions should not be imposed on patients. 3. Compassion and good intentions are not an acceptable basis for actions beyond the scope of nursing practice. 4. These factors do not permit a nurse to be involved with euthanasia.

A student nurse is about to graduate from an accredited nursing program. The student nurse understands that an action unrelated to a state Nurse Practice Act is: 1. Setting guidelines for nurses' salaries in the state 2. Establishing reciprocity for licensure between states 3. Determining minimum requirements for nursing education 4. Maintaining a list of nurses who can legally practice in the state

1. CORRECT: State Nurse Practice Acts define and regulate the practice of nursing within the state. The salary of nurses is determined through negotiations between nurses or their representatives, such as a union or a professional nursing organization, and the representatives of the agency for which they work. 2. A state's Nurse Practice Act determines the criteria for reciprocity for licensure. 3. A state's Nurse Practice Act stipulates minimum requirements for nursing education. 4. A state's Nurse Practice Act defines the criteria for licensure within the state. The actual functions, such as maintaining a list of nurses who can legally practice in the state, may be delegated to another official body such as a State Board of Nursing or State Education Department.

A nurse expert is called to testify in a lawsuit regarding professional nursing malpractice primarily to testify: 1. About standards of nursing care as they apply to the facts in the case 2. With regard to laws governing the practice of nursing 3. For the prosecution 4. For the defense

1. CORRECT: The American Nurses Association Standards of Clinical Nursing Practice are authoritative statements by which the national organization for nursing describes the responsibilities for which its practitioners are accountable. An expert nurse is capable of explaining these standards as they apply to the situation under litigation. These professional standards are one criterion that helps a judge or jury determine if a nurse committed malpractice or negligence. 2. An expert nurse is not an expert in the law. The expert nurse's role is not to make judgments about the laws as they apply to the practice of nursing. 3. A nurse expert can testify for either the prosecution or the defense. 4. A nurse expert can testify for either the defense or the prosecution.

What is the main purpose of the American Nurses Association? 1. Establish standards of nursing practice 2. Recognize academic achievement in nursing 3. Monitor educational institutions granting degrees in nursing 4. Prepare nurses to become members of the nursing profession

1. CORRECT: The American Nurses Association has established Standards of Care and Standards of Professional Performance. These standards reflect the values of the nursing profession, provide expectations for nursing practice, facilitate the evaluation of nursing practice, and define the profession's accountability to the public. 2. Sigma Theta Tau, the international honor society of nursing, recognizes academic achievement. 3. The National League for Nursing Accrediting Commission, the Commission on Collegiate Nursing Education, and state education departments monitor educational institutions granting degrees in nursing. 4. Schools of nursing (diploma, associate degree, and baccalaureate) educate individuals for entry into the practice of nursing.

When choosing a nursing school in the United States that awards an associate degree, a future student nurse should consider schools that have met the standards of nursing education established by which organization? 1. National League for Nursing Accrediting Commission 2. North American Nursing Diagnosis Association 3. American Nurses Association 4. Sigma Theta Tau

1. CORRECT: The National League for Nursing Accrediting Commission (NLNAC) is an organization that appraises and grants accreditation status to nursing programs that meet predetermined structure, process, and outcome criteria. 2. The North American Nursing Diagnosis Association (NANDA) developed a constantly evolving taxonomy of nursing diagnoses to provide a standardized language that focuses on the patient and related nursing care. 3. The American Nurses Association (ANA) is the national professional organization for nursing in the United States. It does not accredit schools of nursing. 4. Sigma Theta Tau, the international honor society of nursing, recognizes academic achievement. It does not accredit schools of nursing.

A Registered Nurse witnesses an accident and assists the victim who has a life-threatening injury. What should the nurse do to meet the most important standard when acting as a Good Samaritan at the scene of an accident? 1. Seek consent from the injured party before rendering assistance 2. Implement every critical-care intervention necessary to sustain life 3. Stay at the scene until another qualified person takes over responsibility 4. Insist on helping because a nurse is the best-qualified person to provide care

1. Depending on the injured person's physical and emotional status, the person may or may not be able to consent to care. 2. When a nurse helps in an emergency, the nurse is required to render care that is consistent with care that any reasonably prudent nurse would provide under similar circumstances. The nurse should not attempt interventions that are beyond the scope of nursing practice. 3. CORRECT: When a nurse renders emergency care, the nurse has an ethical responsibility not to abandon the injured person. The nurse should not leave the scene until the injured person leaves or another qualified person assumes responsibility. 4. A nurse should offer assistance, not insist on assisting, at the scene of an emergency.

Which organization is responsible for ensuring that Registered Nurses are minimally qualified to practice nursing? 1. Sigma Theta Tau 2. State Boards of Nursing 3. American Nurses Association 4. Constituent Leagues of the National League for Nursing

1. Sigma Theta Tau, the international honor society of nursing, recognizes academic achievement and leadership qualities, encourages high professional standards, fosters creative endeavors, and supports excellence in the profession of nursing. This organization does not grant licensure. 2. CORRECT: The National Council of State Boards of Nursing is responsible for the NCLEX examinations; however, the licensing authority in the jurisdiction in which the graduate takes the examination verifies the acceptable score on the examination. 3. The American Nurses Association (ANA) is the national professional organization for nursing in the United States. It fosters high standards of nursing practice; it does not grant licensure. 4. The National League for Nursing (NLN) is committed to promoting and improving nursing service and nursing education; it does not grant licensure.

A faculty member of a nursing program is conducting an informational session for potential nursing students. The faculty member includes the information that at the completion of the program licensure to practice is: 1. A responsibility of the American Nurses Association 2. Granted on graduation from a nursing program 3. Approved by the National League for Nursing 4. Required by state law

1. The ANA Standards of Clinical Nursing Practice do not address licensure. 2. When a person graduates from a school of nursing, the individual receives a diploma that indicates completion of a course of study; the diploma is not a license to practice nursing. 3. The National League for Nursing (NLN) promotes nursing service and nursing education; it is not involved with licensure. 4. CORRECT: The Nurse Practice Act in a state stipulates the requirements for licensure within the state.

The nurse initiates a visit from a member of the clergy for a patient. How is the nurse functioning when initiating this visit? 1. Interdependently 2. Independently 3. Dependently 4. Collegially

1. The nurse does not need a practitioner's order to make a referral to a member of the clergy. An interdependent intervention requires a practitioner's order associated with a parameter. 2. CORRECT: The nurse is initiating the referral to the member of the clergy and is therefore working independently. Nurses are legally permitted to diagnose and treat human responses to actual or potential health problems. 3. This action is within the scope of nursing practice. The nurse does not need a practitioner's order to make a referral to a member of the clergy. 4. The nurse can make a referral to a member of the clergy without collaborating with another professional health-care team member.

When the nurse is administering a medication to a confused patient, the patient says, "This pill looks different from the one I had before." What should the nurse do? 1. Ask what the other pill looked like 2. Explain the purpose of the medication 3. Check the original medication prescription 4. Encourage the patient to take the medication

1. This action by itself is unsafe because the patient is confused and the information obtained may be inaccurate. 2. This intervention ignores the patient's concern. Although this ultimately may be done, it is not the priority action. 3. CORRECT: This is the safest intervention because it goes to the original source of the prescription. 4. This action ignores the patient's statement and is unsafe without first obtaining additional information

The patient's diet order is "clear liquids to regular as tolerated." How is the nurse functioning when progressing the patient's diet to full liquid? 1. Dependently 2. Independently 3. Collaboratively 4. Interdependently

1. This dietary order has parameters that exceed a simple dependent function of the nurse. 2. Prescribing a dietary order for a patient is outside the scope of nursing practice. 3. Collaborative or collegial interventions are actions the nurse carries out in conjunction with other health-care team members. 4. CORRECT: The practitioner's order implies a progression in the diet as tolerated. The nurse uses judgment to determine the time of this progression, which is an interdependent action.

Identify the actions that are examples of slander. Select all that apply. 1. _____ Volunteer telling another volunteer a patient's age 2. _____ Nurse explaining to a patient that another nurse is incompetent 3. _____ Personal care assistant sharing information about a patient with another patient 4. _____ Unit manager documenting a nurse's medication error in a performance appraisal 5. _____ Housekeeper who is angry at a nurse erroneously telling another staff member that the nurse uses cocaine

1. This is a violation of the patient's right to confidentiality, not slander. 2. CORRECT: This is an example of slander. Slander is a false spoken statement resulting in damage to a person's character or reputation. 3. This is a violation of the patient's right to confidentiality, not slander. 4. This is not slander because it is a written, not spoken, statement and it documents true, not false, information. 5. CORRECT: This is an example of slander. It is a malicious, false statement that may damage the nurse's reputation.

The nurse says, "If you do not let me do this dressing change, I will not let you eat dinner with the other residents in the dining room." What legal term is related to this statement? 1. Battery 2. Assault 3. Negligence 4. Malpractice

1. This is not an example of battery. Battery is the actual willful touching of another person that may or may not cause harm. 2. CORRECT: This statement is an unjust threat. Assault is the threat to harm another person without cause. 3. This is not an example of negligence. Negligence occurs when harm or injury is caused by an act of either commission or omission. 4. This is not an example of malpractice. Malpractice is negligence by a professional person as compared with the actions of another professional person in a similar circumstance when a contract exists between the patient and nurse.

11. An 87-year-old woman is brought to the emergency department for treatment of a fractured arm. On assessment, the nurse notes old and new ecchymotic areas on the client's chest and legs and asks the client how the bruises were sustained. The client, although reluctant, tells the nurse in confidence that her son frequently hits her if supper is not prepared on time when he arrives home from work. What is the most appropriate nursing response? a. "Oh really I will discuss this situation with your son" b. "Let's talk about the ways you can manage your time to prevent this from happening" c. "Do you have any friends that can help you out until you resolve these important issues with your son?" d. "As a nurse, I am legally bound to report abuse. I will stay with you while you give the report and help find a safe place for you to stay.

11. D- The nurse must report situations related to child or elder abuse, gunshot wounds and other criminal acts, and certain infectious diseases. Confidential issues are not to be discussed with nonmedical personnel or the client's family or friends without the client's permission. Clients should be assured under a legal obligation. Option 1, 2, and 3 do not address the legal implications of the situation and do not ensure a safe environment for the client.

12. The nurse calls the health care provider (HCP) regarding a new medication prescription because the dosage prescribed is higher than the recommended dosage. The nurse is unable to locate the HCP, and the medication is due to administered. Which action should the nurse take? a. Contact the nursing supervisor b. Administer the dose prescribed c. Hold the medication until the HCP can be contacted d. Administer the recommended dose until the HCP can be located

12. A- If the HCP writes a prescription that requires clarification, the nurse's responsibility is to contact the HCP. If there is no resolution regarding the prescription because the HCP cannot be located or because the prescription remains as it was written after talking with the HCP, the nurse should contact the nurse manager or nursing supervisor for further clarification as to what the next step should be. Under no circumstances should the nurse proceed to carry out the prescription until obtaining clarification.

13. The nurse employed in a hospital is waiting to receive a report from the laboratory via the facsimile (fax) machine. The fax machine activates and the nurse expects the report, but instead receives a sexually oriented photograph. Which is the most appropriate nursing action? a. Call the police b. Cut up the photograph and throw it away c. Call the nursing supervisor and report the incident d. Call the laboratory and ask for the individual's name who sent the photograph

13. C- Ensuring a safe workplace is a responsibility of an employing institution. Sexual harassment in the workplace is prohibited by state and federal laws. Sexually suggestive jokes, touching, pressuring a co-worker for a date, and open displays of or transmitting sexually oriented photographs or posters are examples of conducts that could be considered sexual harassment by another worker. If the nurse believes that he or she is being subjected to unwelcome sexual conduct, these concerns should be reported to the nursing supervisor immediately. Option 1 is unnecessary at this time. Options 2 and 4 are inappropriate initial actions.

2. A client is brought to the emergency department by emergency medical services (EMS) after being hit by a car. The name of the client is unknown, and the client has sustained a severe head injury and multiple fractures and is unconscious. An emergency craniotomy is required. Regarding informed consent for the surgical procedure, which is the best action? a. Obtain a court order for the surgical procedure b. Ask the EMS team to sign the informed consent c. Transport the victim to the operating room for surgery d. Call the police to identify the client and locate the family.

2. C- In general, there are two situations in which informed consent of an adult client is not needed. One is when an emergency is present and delaying treatment for the purpose of obtaining informed consent would result in injury or death to the client. The second is when the client waives the right to give informed consent. Option 1 will delay emergency treatment, and option 2 is inappropriate. Although option 4 may be pursued, it is not the best action

The nurse is caring for a client with stage 4 pancreatic cancer. The patient is alert and oriented times 3 and tells the nurse that he would like to be a DNR/DNI and would like to transition to comfort cares. The patient's family is upset by this and tells the nurse that the patient is depressed and doesn't know what he's saying . What should the nurse do? 1. The nurse should tell the family that she's not getting involved in this family dilemma and will send in the patient's social worker to deal with this matter. 2. The nurse should encourage the client to reconsider his decisions 3. The nurse should honor the patient's decisions 4. The nurse should consult with palliative care and service to have a care conference with the patient to persuade him into changing his mind about stopping treatment

3.

4. The nurse arrives at work and is told to report (float) to the intensive care unit (ICU) for the day because the ICU is understaffed and needs additional nurses to care for the clients. The nurse has never worked in the ICU. The nurse should take which action first? a. Call the hospital lawyer b. Refuse to float to the ICU c. Call the nursing supervisor d. Identify tasks that can be performed safely in the ICU

4. D- Floating is an acceptable legal practice used by hospitals to solve understaffing problems. Legally, the nurse cannot refuse to float unless a union contract guarantees that nurses can work only in a specified area or the nurse can prove the lack of knowledge for the performance of assigned tasks. When encountering this situation, the nurse should set priorities and identify potential areas of harm to the client. The nursing supervisor is called if the nurse is expected to perform tasks that he or she cannot safely perform. Calling the hospital lawyer is a premature action.

5. The nurse who works on the night shift enters the medication room and finds a co-worker with a tourniquet wrapped around the upper arm. The co-worker is about to insert a needle, attached to a syringe containing clear liquid, in the antecubital area. Which is the most appropriate action by the nurse? a. Call security b. Call the police c. Call the nursing supervisor d. Lock the co-worker in the medication room until help is obtain

5. C- Nurse practice acts require reporting impaired nurses. The board of nursing has jurisdiction over the practice of nursing and may develop plans for treatment and supervision of the impaired nurse. This incident needs to be reported to the nursing supervisor, who will then report to the board of nursing and other authorities, such as the police, as required. The nurse may call security if a disturbance occurs, but no information in the question supports this need, and so this is not the appropriate action. Option 4 is an inappropriate and unsafe action.

6. A hospitalized client tells the nurse that a living will is being prepared and that the lawyer will be bringing the will to the hospital today for witness signatures. The client asks the nurse for assistance obtaining a witness to the will. Which is the most appropriate response to the client? a. "I will sign as a witness to your signature." b. "You will need to find a witness on your own.' c. "Whoever is available at the time will sign as a witness for you." d. "I will call the nursing supervisor to seek assistance regarding your request."

6. D- Living wills, also known as natural death acts in some states, are required to be in writing and signed by the client. The client's signature must be witnessed by specified individuals or notarized. Laws and guidelines regarding living wills vary from state to state, and it is the responsibility of the nurse to know the laws. Many states prohibit any employee, including the nurse of a facility where the client is receiving care, from being a witness. Option 2 is nontherapeutic and not a helpful response. The nurse should seek the assistance of the nursing supervisor.

7. The nurse has made an error in a narrative documentation of an assessment finding on a client and obtains the client's record to correct the error. The nurse should take which action to correct the error? a. Documenting a late entry into the client's record b. Trying to erase the error for space to write in the correct data c. Using whiteout to delete the error to write in the correct data d. Drawing one line through the error, initialing and dating, and then documenting the correct information.

7. D- If the nurse makes an error in narrative documentation in the client's record, the nurse should follow agency policies to correct the error. This includes drawing one line through the error, initializing and dating the line, and then documenting the correct information. A late entry is used to document additional information not remembered at the initial time of documentation. Erasing data from the client's record and the use of whiteout are prohibited

9. A nursing instructor delivers a lecture to nursing students regarding the issue of client's rights and asks a nursing student to identify a situation that represents an example of invasion of client privacy. Which situation, if identified by the student, indicates an understanding of a violation of this client right? a. Performing a procedure without consent b. Threatening to give a client a medication c. Telling the client that he or she cannot leave the hospital d. Observing care provided to the client without the client's permission

9. D- Invasion of privacy occurs with unreasonable intrusion into an individual's private affairs. Performing a procedure without consent is an example of battery. Threatening to a give a client a medication constitutes assault. Telling the client that the client cannot leave the hospital constitutes false imprisonment.

A client who insists on leaving the hospital against medical advice (AMA) after being properly advised signs the necessary form. In order to avoid legal consequences regarding the client's decision, the nurse should: A. objectively document all the details of the situation thoroughly but concisely in the client's medical record. B. notify the nursing supervisor of the client's decision and that all reasonable efforts were made to dissuade the client from leaving. C. have a member of the security staff escort the client off the facility's property and into their private vehicle or public transportation D. with the client's permission notify a contact person that the client is leaving the hospital against medical advice.

A Almost all health care facilities have an AMA form that patients are asked to sign when they decide to refuse or discontinue ordered therapy or intend to leave the facility. The value of the document in countering a claim of negligence should the patient or family later sue will depend in great part on the quality of the nurse's charting. Although the other options may be correct procedures, they are not directed towards protecting the nurse legally.

Ethical dilemmas often arise over a conflict of opinions. Each of the following steps constitutes a correct step to take toward resolution of an ethical dilemma. What order should these steps be taken? 1. Clarify your own values about the issue. 2. Call a meeting in which those involved in the dilemma can discuss (negotiate) the possible solutions to the dilemma. 3. State the problem clearly in a way that all involved can understand. 4. Gather all relevant information regarding the clinical, social, and spiritual aspects of the dilemma. A. 4, 1, 3, 2 B. 4, 3, 1, 2 C. 1, 4, 2, 3 D. 1, 4, 3, 2

A The first step to the resolution of any ethical dilemma entails the gathering of all relevant information. In some cases this step may uncover valuable information that resolves the dilemma without further processing. Clarifying values helps you distinguish between your own values and opinions and those of others and the difference between value and fact. Stating the problem clearly ensures efficient discussion as you negotiate possible solutions.

A patient's daughter is speaking to the nurse caring for her father. The daughter has presented the nurse with a document identifying her as the spokesperson for the patient when he is no longer capable of speaking for himself. Which of the following best characterizes the daughter's legal relationship with her ailing father? A. Health care proxy B. Legal Samaritan C. Guardian ad litem D. Attorney

A The health care proxy is a surrogate decision maker chosen by the patient and documented in a legal form dated and signed by the patient and notarized by an impartial notary (in many states). Formally this document is known as a durable power of attorney for health care. There is no such characterization known as a legal Samaritan. A guardian ad litem is a person chosen by a court of law with written documentation describing the guardian status. A guardian ad litem is not limited to health care decisions. An attorney may or may not be a health care proxy. A health care proxy does not have to be an attorney.

Utilitarianism is a term commonly found in ethical discourse, but it stands for only one of several different approaches to ethical discourse. Which is a true statement about the ethical philosophy of utilitarianism? A. The value of an intervention is determined primarily by its usefulness to society. B. The value of an intervention is culturally established based on predetermined measures. C. The decision to provide medical care depends on a measure of the moral life of the patient. D. Attention to relationships provides resolution to ethical dilemmas.

A Utilitarianism is based on the notion of "usefulness." "The value of an intervention is culturally established based on predetermined measures" and "Attention to relationships provides resolution to ethical dilemmas" describe other philosophical approaches to ethical discourse but not utilitarianism. "The decision to provide medical care depends on a measure of the moral life of the patient" is a false statement.

c,d,e (Rationale The elements that need to be included are the exact location in the facility of the​ event, the identification of equipment involved in the​ event, and the name of the nurse completing the report. The names of all staff on duty at the time of the event are not recorded. All witnesses to the​ event, whether or not they are​ clients, are included.)

A nurse is filling out an incident report for the first time. The nurse wants to make sure that the report is complete. What elements must be included to give the best picture of the​ event? ​(Select all that​ apply.) a Witnesses to the​ event, except for other clients b Names of all staff on duty at the time of the event c The name of the nurse completing the report d The identification of equipment if involved in the event e The exact location in the facility of the event

At the hospital where you work, you care for a child admitted frequently for management of cystic fibrosis. The child's family has initiated a Cystic Fibrosis Support Group page on Facebook, and they invite you to "friend" their page. Which of the following justifications would you use to explain your decision to accept or not accept the invitation? (Select all that apply.) A. Nurse-patient boundaries may be violated, harming possibility for therapeutic relationship. B. By accepting you could share nursing information online about the patient as a way to educate the support group. C. Postings can easily spread to a wider audience with the potential for HIPAA violations. D. The law prohibits your use of social networking with patients.

A, C Participation in online social media such as Facebook entails hazards, such as nurse-patient boundaries being violated or postings being viewed by a much wider audience with potential for HIPAA violations. These justify declining an invitation to "friend" a patient's support group online. Laws do not yet exist prohibiting nurse-patient social networking, although most health care facilities have established policies that guide participation in social media. The hazards involved in a nurse's participation in social media may not be obvious. Hazards include the interruption of relationships of trust and therefore therapeutic relationships. Postings can be shared with others outside the privacy limits established by the original user. Images and information posted innocently may violate privacy when an unintended audience views the page.

Ethics in nursing practice includes an embrace of accountability or the ability to justify your actions. Even though your practice is defined in part by orders written by health care providers and policies enforced by administrators, you remain ethically accountable for your actions. Which of the following actions illustrates accountability? (Select all that apply.) A. Your patient receives a surgical procedure that is new to your facility. You ask your manager to provide an in-service about the procedure. B. A health care provider writes orders for pain-management medication even though the patient has been free of pain for 3 days. Out of respect for the health care provider's legal responsibilities, you administer the medications. C. During annual budget preparation at your facility, you advocate for annual pay increases for you and your peers. D. Your patient confides in you that she has recently lost her job and is anxious about her medical bills, including her ability to pay for medications after discharge. Health care coverage is not your area of expertise, but you know that the social worker might be able to help. You initiate a consultation request.

A, D Taking accountability for actions involves acting independently and taking initiative to remain competent and in taking initiative to act in the patient's best interest. Declining to question a health care provider's orders represents a lack of personal accountability. Asking for a pay raise is more about advocacy than accountability.

A nurse is instructing a group of students about how to know and what to expect when ethical dilemmas arise. Which of the following situations should the students identify as an ethical dilemma? A. A nurse on a medical-surgical unit demonstrates signs of chemical impairment. B. A nurse overhears another nurse telling an older adult client that if he doesn't stay in the bed, she will have to apply restraints. C. A family has conflicting feelings about the initiation of enteral tube feedings for their father, who is terminally ill. D. A client who is terminally ill hesitates to name her spouse on her durable power of attorney form.

A. INCORRECT: Delivering client care while showing signs of a substance use disorder is a legal issue, not an ethical dilemma. B. INCORRECT: A nurse who threatens to restrain a client has committed assault. This is a legal issue, not an ethical dilemma. C. CORRECT: Making the decision about initiating enteral tube feedings is an example of an ethical dilemma. A review of scientific data cannot resolve the issue, and it is not easy to resolve. The decision will have a profound effect on the situation and on the client. D. INCORRECT: The selection of a person to make healthcare decisions on a client's behalf is a legal decision, not an ethical dilemma.

An adult client who is competent tells the nurse that he is thinking about leaving the hospital against medical advice. The nurse believes that this is not in the client's best interest, so she administers a PRN sedative medication the client has not requested along with his usual medication. Which of the following types of tort has the nurse committed? A: Assault B: False imprisonment C: Negligence D: Breach of confidentiality

A: INCORRECT: Assault is an action that threatens harmful contact without the client's consent. The nurse has made no threats in this situation. B: CORRECT: The nurse gave the medication as a chemical restraint to keep the client from leaving the facility against medical advice. C: INCORRECT: Negligence is a breach of duty that results in harm to the client. It is unlikely that medication that the nurse administered without his consent actually harmed the client. D: INCORRECT: The nurse has not disclosed any protected health information, so there is no breach of confidentiality involved in this situation.

A nurse has noticed several occasions in the past week when another nurse on the unit seemed drowsy and unable to focus on the issue at hand. Today, she found the nurse asleep in her chair in the break room when she was not on a break. Which of the following actions should the nurse take? A: Remind the nurse that safe client care is a priority on the unit. B: Ask others on the team whether they have observed the same behavior. C: Report her observations to the nurse manager on the unit. D: Conclude that her coworker's fatigue is not her problem to solve.

A: INCORRECT: Confronting the coworker might cause her to respond defensively and does nothing to resolve the problem. B: INCORRECT: Finding out whether others have noticed the problem is immaterial and should not affect the nurse's course of action. C: CORRECT: Any nurse who notices behavior that could jeopardize client care or could indicate a substance use disorder has a duty to report the situation immediately to the nurse manager. D: INCORRECT: The nurse may not be responsible for solving the problem, but she does have a duty to take action since she has observed the problem.

A nurse is instructing a group of nursing students about the responsibilities involved with organ donation and procurement. When the nurse explains that all clients waiting for a kidney transplant have to meet the same qualifications, the students should understand that this aspect of care delivery is and example of which of the following ethical principles? A. Fidelity B. Autonomy C. Justice D. Nonmaleficence

A: INCORRECT: Fidelity is an agreement to keep promises. Because donor organs are a scarce resource compared with the numbers of potential recipients who need them, no one can promise anyone an organ. Thus, this principle does not apply. B: INCORRECT: Autonomy is the right to make personal decisions, even when they are not necessarily in the person's best interest. No personal decision is involved with the qualifications of organ recipients. C: CORRECT: Justice is fairness in care delivery and in the use of resources. By applying the same qualifications to all potential kidney transplant recipients, organ procurement organizations demonstrate this ethical principle in determining the allocation of these scarce resources. D: INCORRECT: Nonmaleficence is the avoidance of harm or injury. In this situation, harm can occur to organ donors and to recipients. The requirements of the organ procurement organizations are standard procedures and do not address avoidance of harm or injury.

A client who will undergo neurosurgery the following week tells the nurse in the surgeons's office that he will prepare his advance directives before he goes to the hospital. Which of the following statements by the client indicates to the nurse that he understands advance directives? A: "I'd rather have my brother make decisions for me, but I know it has to be my wife." B. "I know they won't go ahead with surgery unless I prepare these forms." C. "I plan to write that I don't want them to keep me on a breathing machine." D. "I will get my regular doctor to approve my plan before I hand it in at the hospital."

A: INCORRECT: The client may designate any competent adult to be his health care proxy. It does not have to be my spouse. B: INCORRECT: Although the hospital staff must ask the client whether he has prepared advance directives and provide written information about them if he hasn't, they may not refuse care based on the lack of advance directives. C: CORRECT: The client has the right to decide and specify which medical procedures he wants when a life-threatening situation arises. D: INCORRECT: The client does not need his provider's approval to submit his advance directives. However, he should give his primary care provider a copy of the document for his records.

A client with acquired immune deficiency syndrome (AIDS) is admitted to the acute care floor. According to a 2015 American Nurses Association (ANA) position statement, which stance addressing this bioethical issue is appropriate? A) The nurse is morally obligated to care for the client unless the risk exceeds responsibility. B) The nurse has the responsibility to ensure the client gets adequate medical care. C) The client has the right to choose not to disclose his or her condition to staff. D) The client is morally bound to disclose every aspect of his or her condition to staff.

Answer: A Explanation: A) AIDS continues to bear a social stigma in our society because of its association with illicit drug use and sexual behavior. According to a 2015 ANA position statement, the nurse cannot set aside the moral obligation to care for the client infected with human immunodeficiency virus (HIV) unless the risk exceeds the responsibility.

A hospice nurse is working closely with a client who, on several occasions, has asked about guidance and support in ending life. The nurse recognizes which in regard to making ethical and moral decisions in this circumstance? A) Euthanasia has legal implications along with moral and ethical ones. B) Passive euthanasia is an easy decision to arrive at. C) Active euthanasia is supported in the Code for Nurses. D) Assisted suicide is illegal in all states.

Answer: A Explanation: A) Determining whether an action is legal is only one aspect of deciding whether it is ethical. Legality and morality are not one and the same. The nurse must know and follow the legal statutes of the profession and boundaries within the state before making any decision. Passive euthanasia involves the withdrawal of extraordinary means of life support and is never an easy decision. Active euthanasia and assisted suicide are in violation of the Code for Nurses, according to the position statement by the ANA (2013). Some states and countries have laws permitting assisted suicide for clients who are severely ill, are near death, and wish to commit suicide.

An adolescent client diagnosed with leukemia decides to stop chemotherapy treatments. The parents of the client, however, want the healthcare team to continue all treatments as necessary. Which action by the nurse is appropriate when providing care to this client and family? A) Helping the family by providing information and allowing them to voice concerns B) Confronting the parents and telling them not to be "selfish" in their child's time of need C) Calling the authorities immediately D) Obtaining a court order to determine the client legally able to make his or her own decisions

Answer: A Explanation: A) Parents have the authority to make healthcare decisions for their children. Dilemmas arise when parents and children do not agree on whether or not to go forward with a recommended treatment. In most cases, the nurse and other members of the healthcare team who have developed a therapeutic alliance with the child and family may be able to help the family come to a joint decision by providing additional information and opportunity to discuss their concerns with each other calmly and openly. In some cases, however, the healthcare team may need to seek guidance from the agency's ethics committee.

While making rounds on the night shift, a nursing supervisor notes that a 73-year-old client under observation following a myocardial infarction has multiple visible bruises on the arms and legs. The supervisor suspects abuse because nothing in the client's chart suggests this client should have sustained these injuries. This state's good faith immunity applies in cases of suspected abuse not only of children but also of older adults or adults with disabilities. Which action has the highest priority for the nursing supervisor in this situation? A) Notify authorities regarding the suspected abuse. B) Do nothing about the situation. C) Notify the security department. D) Ask a shift nurse about the source of the injuries.

Answer: A Explanation: A) States also have specific laws pertaining to the mistreatment of adults and older adults. These laws may be similar to those that govern the abuse and neglect of children. For example, many states generally offer good faith immunity to individuals who report suspected abuse or neglect of an older adult or an adult with a disability. This immunity would apply in the case of this client. Security is not the appropriate department to notify unless the injuries were sustained at work. Questioning a shift nurse about the source of the injuries is fine to do but should not replace reporting the injuries to the appropriate authorities, which should be done in this case because the supervisor suspects abuse.

Which of the following advocacy interventions might nurses provide that are within their role and scope of practice? Select all that apply. A) Educating clients and their families about their legal rights regarding informed decision-making B) Ensuring that clients have the necessary information to make an informed decision or give informed consent C) Evaluating organizational policies and procedures to ensure protection of client rights D) Supporting medical authority even when this goes against the client's wishes E) Declaring clients incompetent so family members can make medical decisions for them

Answer: A, B, C Explanation: A) Clients must understand their rights in order to be able to defend them. As an advocate, the nurse provides clients with the information they need to make informed decisions and supports the clients' rights to make their own healthcare decisions. Nurses should evaluate organizational policies and procedures and monitor clients' care to ensure protection of client rights. A nurse should understand that advocacy may require political action. Conflicts may arise over issues that require consultation, confrontation, or negotiation between the nurse and administrative personnel or between the nurse and primary care providers. Declaring clients incompetent is not the role of the nurse, and it should not be done just to please family members.

A nurse working on a medical-surgical unit wants to ensure care is provided within the standard of nursing care. Which actions by the nurse are appropriate? Select all that apply. A) Analyze the position description. B) Review and become familiar with the policy and procedure manual. C) Question the value of collaborating with other disciplines. D) Review applicable state nurse practice act and administrative rules. E) Adhere to national standards of practice and care.

Answer: A, B, D, E Explanation: A) Nurses are expected to demonstrate competence within multiple areas of their professional role, including collaboration with the entire care team. The nurse's specific job description will contribute to defining the standard of care. Employers can limit but not expand the scope of practice, and the nurse will be held to functioning within the scope of employment. Agency policies and procedures serve in defining the standard of care. The applicable state nurse practice act and administrative rules form the basis of the standard of care to which each nurse is held. A primary source for defining the standard of care is the prevailing national nursing standards. Nurses who follow national standards of practice and standards of care will provide their clients with the best care possible and be far less likely to commit any unintentional act that may rise to the level of malpractice.

The nurse is concerned about being sued for negligence when providing care. Which nursing actions may be grounds for negligence? Select all that apply. A) Client fell getting out of bed because the call light was not used. B) Client name band was checked prior to providing all medications. C) Client's morning medications were administered in the early afternoon. D) Client states not understanding activity restrictions and wound eviscerated. E) Client documentation did not include appearance of infiltrated IV site.

Answer: A, C, D, E Explanation: A) Checking the client name band before providing medications is not an action that is negligent. However, providing medications beyond the prescribed time can be viewed as negligent care. One strategy to prevent instances of professional negligence is to ensure client safety. The client fell when getting out of bed because the call light was not used. Because there is no way of knowing if the client knew how to use the call light, the nurse should be concerned with this situation. Clear communication of directions, explanations, and providing effective client education regarding the client's healthcare requirements can help decrease the risk of bad outcomes, so the wound evisceration could be viewed as negligent care. Poor documentation about care, wounds, and intravenous sites could be viewed as negligent care.

A school nurse who is concerned about an increase in sports injuries related to ineffective protective equipment decides to hold a community seminar related to the importance of proper protective equipment. Which advocacy activities is this nurse demonstrating? Select all that apply. A) Advocating for vulnerable populations B) Advocating for fair and equitable access to high-quality care for all clients C) Ensuring that clients have the necessary information to make an informed decision or give informed consent D) Informing the public about issues and concerns E) Speaking publicly for the health, welfare, and safety of their clients

Answer: A, C, D, E Explanation: A) Clients from vulnerable populations, such as children, particularly benefit from nursing advocacy. Through this seminar, the nurse is providing clients with information to make an informed decision and informing the public about issues and concerns that are important to them. The nurse is also speaking publicly to encourage safety practices for students. These are all important advocacy activities. Advocating for fair and equitable access to high-quality care is also an important advocacy activity, but it is not being demonstrated in this situation.

11) A school nurse is particularly concerned about the sports injuries related to ineffective protective equipment. She decides to participate in a child fatality prevention committee. What are the advocacy activities this nurse is demonstrating? Select all that apply. A) Advocating for vulnerable populations B) Advocating for fair and equitable access to high-quality care for all clients C) Ensuring that clients have the necessary information to make an informed decision or give informed consent D) Informing the public about issues and concerns E) Speaking publicly for the health, welfare, and safety of their clients

Answer: A, C, D, E Explanation: A) Clients from vulnerable populations, like children, particularly benefit from nursing advocacy. Although advocating for fair and equitable access is an important advocacy activity, it is not part of the work of a fatality prevention committee. A child fatality prevention committee may recognize and act on information early. This type of committee may be able to recognize patterns and devise related health promotion activities. As advocates, both professional and public, nurses can speak to promote and effect change.

The nurse is caring for a 22-year-old client with Down syndrome. Because the client has an intellectual disability, he is under the legal care of his parents. The client needs medical treatment for aspiration pneumonia, but the parents are declining care because they have heard that aspiration pneumonia is often fatal in clients with chronic health conditions. In addition to ethics and advocacy, what other nursing concept must the nurse factor into care decisions made in this case? A) Informatics B) Development C) Mood and Affect D) Spirituality

Answer: B Explanation: A) Down syndrome causes intellectual disability, so the client's developmental stage needs to be taken into consideration when providing care, especially related to client teaching and advocating for the client whose rights appear to be in jeopardy. Informatics, mood and affect, and spirituality do not appear to play a role in this case.

An adolescent client with a sexually transmitted infection (STI) says to the nurse, "Promise you won't tell my parents about my condition." The agency policy is that all STIs must be reported in accordance with federal and state law. Which action by the nurse is appropriate? A) Disclosing information to the parents B) Reporting the STI to the proper authorities C) Respecting the client's privacy and confidentiality by not mentioning or reporting the STI D) Telling other nurses in the clinic that the client has an STI

Answer: B Explanation: A) In this case, the nurse is required to report information about the client's STI to the state health department. Because of confidentiality issues, the nurse should not report the STI to the parents or to other nurses not involved in the client's care.

Which statement accurately describes the purpose of the American Nurse's Association's Code of Ethics for Nurses? A) It serves as a statement of nurses' personal values and standards. B) It serves as the profession's nonnegotiable ethical standard. C) It serves as an announcement of nurses' commitment to the profession. D) It serves as a standard protocol for performing nursing procedures.

Answer: B Explanation: A) The ANA Code of Ethics for Nurses serves as a statement of nurses' ethical obligations and duties (not their personal values and standards), as the profession's nonnegotiable ethical standard, and as the nursing profession's statement of commitment to society (not the nurse's commitment to the profession). Nurses should refer to the ANA Code of Ethics for Nurses to direct how they perform their duties in daily practice, but it does not provide standard protocols for performing nursing procedures.

A charge nurse notices that a client has a black eye that was not present when admitted to the facility. Which action by the charge nurse is appropriate in this situation? A) Ask a staff nurse to question the client about the situation. B) Discuss the situation with the client in a private setting. C) Ask the other staff members if abuse is involved. D) Ignore the situation until the client shows a willingness to talk.

Answer: B Explanation: A) The charge nurse should discuss the situation with the client in private and offer options of help. The charge nurse should not ignore the situation and should advocate for the client. The charge nurse herself should address this situation. The nurse should speak to the client first, not the staff, and not assume abuse until the client has given her version of events.

By providing volunteer client care to an inadequately insured population, the nurse is demonstrating which value of client advocacy? A) The client has the right to make choices and decisions. B) The nurse has the responsibility to ensure the client has access to healthcare services. C) The client has the right to expect a nurse-client relationship based on shared respect. D) The nurse has the responsibility to make choices and decisions.

Answer: B Explanation: A) The nurse has the responsibility to ensure the client has access to healthcare services that meet health needs. Although the client does have the right to make choices and decisions, the nurse volunteering at a free clinic to provide healthcare to the underinsured does not demonstrate this value. Although the client does have the right to expect a nurse-client relationship based on shared respect, the nurse volunteering at a free clinic to provide healthcare to the underinsured does not demonstrate this value. The nurse's responsibility to make choices and decisions is not one of the values basic to client advocacy.

The nurse is talking with a parent who decides to decline treatment for a 3-year-old client whose cancer has metastasized. There is a conflict between the parents and the rest of the family regarding the withdrawal of care from the child. Which should the nurse consider when determining the appropriate action for this client? A) The beliefs of the child B) The values of the parents C) The age of the child D) The values of the rest of the family

Answer: B Explanation: A) When confronted with a conflict regarding care, one of the first actions by the nurse is to consider the values and beliefs of the parents who are making the decision. The age of the child is not a relevant factor in the decision making if the child is under 18 years. The child is too young to have values and beliefs. The nurse is respectful with the rest of the family but should consider the parents' decision only.

3) A charge nurse notices that a coworker who is assigned to client care has come to work with a black eye. The charge nurse is aware that the nurse lives with her boyfriend, who has caused problems in the past. What is the best action by the charge nurse? A) Notify security to approach the nurse about the situation. B) Discuss the situation with the nurse in a private setting. C) Ask the other staff members if abuse is involved. D) Ignore the situation until the nurse shows a willingness to talk.

Answer: B Explanation: A) The charge nurse should discuss the situation with the nurse in private and offer options of help. The charge nurse should not ignore the situation and should advocate for the nurse. Security cannot address situations that do not occur on hospital grounds. It would be a breach of confidentiality to talk to the rest of the staff.

2) A nurse is volunteering at a local free clinic. By providing client care to an inadequately insured population, the nurse is demonstrating which value of client advocacy? A) The client has the right to make choices and decisions. B) The nurse has the responsibility to ensure the client has access to healthcare services. C) The client has the right to expect a nurse-client relationship based on shared respect. D) The nurse has the responsibility to make choices and decisions.

Answer: B Explanation: A) The nurse has the responsibility to ensure the client has access to healthcare services that meet health needs. Although the client does have the right to make choices and decisions, the nurse volunteering at a free clinic to provide health care to the underinsured does not demonstrate this value. Although the client does have the right to expect a nurse-client relationship based on shared respect, the nurse volunteering at a free clinic to provide health care to the underinsured does not demonstrate this value. The nurse's responsibility to make choices and decisions is not one of the values basic to client advocacy.

An older adult client with metastasized breast cancer informs the nurse that her doctor is insisting that she participate in a course of chemotherapy, even though the client does not want to have any further treatment. Which actions by the nurse exemplify advocacy for this client? Select all that apply. A) Tell the client that it is in her best interest to follow the doctor's advice. B) Inform the doctor about the client's clear wishes not to have further chemotherapy. C) Ascertain whether or not the client has an advance care directive and, if not, assist her in creating one. D) Discuss the implications of various choices with the client. E) Avoid interfering in the doctor-client relationship.

Answer: B, C, D Explanation: A) Nurses acting as advocates should honor the moral principles and standards and respect clients' right to make their own choices. The nurse should continuously advocate for the client in a professional manner. The nurse serves as both a teacher and an advocate by informing clients about their rights. When the client makes decisions about his or her treatment other than what is recommended, it is the nurse's role to ensure that the client is making an informed decision and, if so, to advocate for the client's right to make autonomous choices.

8) An 83-year-old woman with metastasized breast cancer informs the nurse that her doctor is insisting that she participate in a course of chemotherapy, even though the client does not want to have any further treatment. How can the nurse function as an advocate? Select all that apply. A) Tell the client that it is in her best interest to follow the doctor's advice. B) Inform the doctor about the client's clear wishes not to have further chemotherapy. C) Ascertain whether or not the client has an advanced care directive and, if not, assist her in creating one. D) Discuss the implications of various choices with the client. E) Avoid interfering in the doctor-patient relationship.

Answer: B, C, D Explanation: A) Nurses acting as advocates should honor the moral principles and standards and respect clients' right to make their own choices. The nurse should continuously advocate for the client in a professional manner. The nurse serves as both a teacher and an advocate by informing clients about their rights. When the client makes decisions about his or her treatment other than what is recommended, it is the nurse's role to ensure that the client is making an informed decision and, if so, to advocate for the client's right to make autonomous choices.

A nurse suspects another healthcare provider is under the influence of alcohol at work. Which actions by the nurse are correct? Select all that apply. A) Assume that healthcare provider is handling any problem. B) Immediately report it to a supervisor. C) Respect the privacy of the healthcare provider. D) Assist the provider while care is provided to clients. E) Follow the state board guidelines.

Answer: B, E Explanation: A) A nurse who suspects a colleague of engaging in illegal, immoral, or unethical conduct and fails to act is in direct violation of the ANA Code of Ethics for Nurses. Nurses have a legal responsibility to report any professional whom they suspect of engaging in illegal, immoral, or unethical activities. Although an impaired healthcare provider may view this intervention as an invasion of privacy, such prompt action will safeguard the client from harm, at the same time offering the impaired healthcare provider a chance at recovery. Impairment of a coworker or team member is the most common situation encountered by healthcare professionals. Nurses should follow guidelines set forth by the board of nursing for the state in which they work.

10) An OR nurse notices that the nurse-anesthesiologist for her next scheduled surgery seems to be unsteady. When she comes close to the anesthesiologist, she is aware of a strong odor of alcohol. What should the nurse do? Select all that apply. A) Assume that the surgeon is handling any problem. B) Immediately report it to a supervisor. C) Respect the privacy of the anesthesiologist. D) Decide that a professional coworker is unlikely to come to the OR impaired. E) Follow the state board guidelines.

Answer: B, E Explanation: A) A nurse who suspects a colleague of engaging in illegal, immoral, or unethical conduct and fails to act is in direct violation of the ANA Code of Ethics for Nurses. Nurses have a legal responsibility to report any professional whom they suspect of engaging in illegal, immoral, or unethical activities. Although an impaired healthcare provider may view this intervention as an invasion of privacy, such prompt action will safeguard the client from harm, while at the same time offer the impaired healthcare provider a chance at recovery. Impairment of a coworker or team member is the most common situation encountered by healthcare professionals. Nurses should follow guidelines set forth by the board of nursing for the state in which they work.

A pregnant client with a history of drug use refuses testing for human immunodeficiency virus (HIV), despite the recommendation of her nurse-midwife. Which actions by the nurse are appropriate in this situation? Select all that apply. A) Refuse to treat the client. B) Do the testing anyway. C) Re-emphasize the importance of HIV testing. D) Offer counseling. E) Throughout the pregnancy, encourage the client to reconsider the decision not to be tested.

Answer: C, D, E Explanation: A) Refusing HIV testing is legal and is not mandatory for obtaining care. However, if the mother-to-be is HIV-positive, the test will help her protect her health and the health of her child by obtaining appropriate treatment. The nurse should emphasize the importance of HIV testing and encourage her to receive the test. Offering counseling would also be appropriate. Refusing to treat the client is against the ANA position statement on risk and responsibility in nursing. Doing the test anyway goes against the client's right to autonomy.

A 16-year-old client has requested that she be examined and receive counseling without her parents being present. Which response demonstrates a correct response to this request? A) The nurse asks the client's parents if this is okay with them. B) The nurse agrees but still informs the parents immediately of everything they did not witness. C) The nurse strongly urges the client to reconsider this request to receive the best possible care. D) The nurse agrees that the client has the right to make this request but suggests that the parents still be present and involved.

Answer: D Explanation: A) Adolescent clients may wish to be examined or receive counseling separate from their parents. The nurse should make every effort to honor this request, though doing so may lead to confrontation with the parents. Understanding state statutes and organizational policy related to adolescent confidentially is essential when situations such as this arise. When providing confidential care to adolescents, the nurse should encourage adolescents to consider involving parents or guardians in their decision making. The nurse should make it clear that this is a suggestion and not a requirement for receiving care. The nurse should not clear this request with the parents, involve the parents anyway, or make it sound as though competent care depends on the adolescent reconsidering her request.

Which of the following statements describes the nurse's duty to investigate suspected abuse of a pediatric client before reporting it? A) The nurse must question a parent or guardian about the suspected abuse. B) The nurse must personally observe the client being abused. C) The nurse must identify at least two witnesses who will testify that the client was abused. D) The nurse does not need to investigate suspected abuse of a pediatric client.

Answer: D Explanation: A) Regardless of the situation, the nurse is not required to conduct any type of investigation or otherwise confirm that the suspected abuse of a pediatric client has, in fact, occurred. The nurse is required only to have a good faith suspicion based on information disclosed by a client, physical symptoms observed in a client, or the nurse's personal observations of behavior on the part of a client, colleague, or third party. The nurse is not required, therefore, to question a parent or guardian about the abuse, personally observe the client being abused, or identify witnesses who will testify to the abuse.

The nurse has been working in a long-term care facility for 1 week. The nurse notes that during the evening meal, an unlicensed assistive personnel (UAP) gives a tray to a client who is unable to cut up and eat the food independently, and then leaves. After the nurse assists the client with eating the meal, which action is appropriate to advocate for this client? A) Report the UAP for neglect. B) Notify the healthcare provider. C) Call the client's family to have them assist with evening meals. D) Discuss the situation with the director of nursing.

Answer: D Explanation: A) The nurse would advocate getting the client's plan changed because the goal is to have someone available to help the client eat for every meal. Notifying the doctor will not help the client. The family might be able to help at times but cannot be expected to come for every meal. The nurse assesses that this happens at every meal and seeks to change how this client is cared for, not just changing one healthcare worker. The UAP is not neglecting the client. The UAP is assigned tasks by the nurse in charge of the client.

While administering a medication, the patient tells the nurse that the pill taken at home does not look like this one. Which is the best action taken by the nurse? A. Asking the patient to bring in their own medication to take while they are in the hospital B. Completing an incident report C. Holding the medication and documenting that the patient refused it D. Explaining to the patient the pill is from a different manufacturer after verifying the medication is correct

Answer: D If a patient questions a medication, the nurse should review the order and verify that the medication is correct. Once the order and medication are verified, the nurse should explain to the patient that the pill itself may look different because it is supplied by a different manufacturer. It is not appropriate to hold the medication or to return a medication to the pharmacy, unless it has been confirmed that it is not the correct medication. It is not appropriate to ask a patient to provide their own medication, unless the hospital pharmacy has approved this. Holding and documenting a medication because a patient has questioned it is not appropriate—the medication and order must be verified and validated.

Which of the following is the best example of a situation where a nurse could demonstrate workforce advocacy? A. A client wishes to become an organ donor in spite of family opposition. B. A client feels pressured by a health care provider into agreeing to a specific medical procedure. C. A nurse volunteers time to participate in a fund raiser to provide toys for a hospital's pediatric unit. D. Staff on a particular unit feel that the nurse manager is showing favoritism when granting vacation time.

B Nurses' strong concern and commitment to patient care and their role as patient advocates often place them in direct conflict with those who have more control, such as physicians and health care administrators. How a nurse reacts to such conflicts within the workplace and continues to advocate to improve patient care is a necessary focus for the profession—a focus called workforce advocacy. Volunteering time to provide funds for toys shows an interest in the client's welfare but is not as strongly related to client care. Advocacy for a client wishing to become an organ donor is an example of patient advocacy. The issue of fair management in this situation does not involve client care.

You have agreed to serve on a Policy and Procedure committee at your hospital, representing the voice of bedside nurses from your unit. The committee is discussing a revision to the staffing ratio policy at your hospital by discussing these questions: How many patients can a nurse safely and effectively care for on your unit? Does the ANA professional code of ethics support your concerns about staffing ratios? Indicate the best answer. A. No, the code describes philosophical principles that are important to ethical discourse but unrelated to staffing ratios. B. Yes, the code supports nurses' participation in conditions of employment, including the promotion of quality health care using both individual and collective action. C. No, to support staffing ratio discussions the ANA publishes journals containing research about best practices in a variety of health care settings D. No, the code is not necessary for this discussion since historical foundations of nursing as defined by Florence Nightingale established staffing ratios before the ANA code of ethics.

B The code of ethics clearly and convincingly establishes guidelines to shape nursing practice by promoting ethical behaviors, including the promotion of nursing involvement in issues that shape nursing practice beyond individual behaviors.

A nurse has received a letter from the State Board of Nursing in which he practices that he has been placed on probation and that his license is suspended. The nurse has received no other information about this action. What, if any, claims does this nurse have? A. The nurse has a claim against the State Board of Nursing for a violation of his right to privacy. B. The nurse has a claim against the State Board of Nursing for violating his rights to due process. C. The nurse has a claim against the hospital where he works for failing to represent him in a civil law suit. D. The nurse has no claim against the State Board of Nursing and the hospital.

B The nurse has a claim against the State Board of Nursing. He has a right of due process, which includes the right to timely and sufficient notice of the charges against him. He has a right to a hearing regarding the charge, the right to review evidence against him or her, and the right to question those who bring evidence against him. There is no violation of the nurse's privacy in the current fact pattern. The nurse's hospital may or may not be required to represent the nurse, depending on the nature of the charge and whether it related to the scope of the nurse's employment.

Your adult patient is scheduled for an x-ray film of the head. He is refusing to go, despite the fact that the x-ray film will give vital information related to his chief complaint of a headache. The nurse learns of the patient's refusal and comes in to the patient's room saying, "If you don't go to this x-ray, I'll have to give you a shot to put you out." In your opinion, has the nurse committed a legal mistake? A. No, the nurse is acting in the best interests of the patient who needs the test to treat him. B. No, the nurse is merely trying to help the patient understand the necessity of cooperating with the ordered treatment regimen. C. Yes, the nurse may have committed an assault on the patient by verbally threatening him. D. Yes, the nurse may have committed malpractice by forcing the patient to do something against his will.

C An assault is any verbal or nonverbal threat that places the recipient in reasonable fear of imminent danger. Stating that the nurse will give the patient an injection to make him comply with a treatment is a verbal threat. If the patient is reasonably afraid of getting the x-ray film and the injection to sedate him if he continues to refuse, he may have a reasonable fear of imminent harm or danger. Although the x-ray film may be beneficial to the patient, the patient is an adult with apparent sufficient capacity to refuse a treatment. Therefore an argument related to his "best interest" has no relevance to his refusal. The issue of malpractice does not exist unless and until the nurse follows through on the threat, at which point the nurse will have breached her duty to the patient to respect his right to self-determination/autonomy.

You are working in an intensive care unit on the night shift. You have been caring for the same patient for three nights in a row. The patient's mother sleeps at the patient's bedside. Over time the mother has come to trust you, as evidenced by her long conversations with you while her child sleeps. Earlier in the week, in the presence of health care providers during morning rounds, she consented to an experimental surgical intervention for her child. But in conversation with you, she shares her doubts and confusions about the intervention. In the morning you ask the health care provider to consider an ethical consultation. What is the value of this nurse participating in discussions about ethical dilemmas? (Select all that apply.) A. Most state laws require that ethics committees include a nurse representative. B. The principal of beneficence promotes kindness in nurses. C. Nurses provide unique insight about patients that can be critical to the resolution of ethical dilemmas. D. Nurses can help articulate a patient's point of view based on specific nursing knowledge. E. Health care providers generally do not participate in ethical discourse.

C, D Nurses assess patients in ways that are unique to nursing. Furthermore they often find opportunity for communication with patients that differs in quantity and quality from other health care providers. As a result, nurses obtain information that other professionals may not notice or appreciate

A 16-year-old client has requested that she be examined and receive counseling without her parents being present. Which response demonstrates a correct response to this request? A) The nurse asks the client's parents if this is okay with them. B) The nurse agrees but still informs the parents immediately of everything they did not witness. C) The nurse strongly urges the client to reconsider this request to receive the best possible care. D) The nurse agrees that the client has the right to make this request but suggests that the parents still be present and involved.

D

According to annual assessments performed by the Federal Government, certain groups of people in the United States have poor or no access to health care. You decide to write an editorial to your local newspaper expressing your opinion about this situation. Which ethical principle would you incorporate into your editorial? A. Accountability because as the nurse you are accountable for the well-being of all patient groups B. Respect for autonomy because autonomy is violated if care is not accessible C. Ethics of care because the caring action would be to provide resource access for all D. Justice since this concept addresses questions about the fair distribution of health care resources

D Accountability, respect for autonomy, and ethics of care are valuable and could be incorporated into a discussion about access, but the fundamental principle that shapes thinking about access to care is the principle of justice.

A patient is discussing her surgery with her surgeon. The physician leaves and asks you to have the patient sign the consent form in a few hours. Which statement made by the patient indicates that informed consent has likely been achieved? A. The patient states that the doctor has told her there is nothing more they can do and she is going home. B. The patient states that she has not spoken with her surgeon at all today. C. The patient states that her surgeon has told her that she doesn't need surgery. D. The patient states that she is having surgery on her leg in the morning and that she will have some pain and bleeding for a few days.

D By stating that she is having surgery in the morning and that she understands some pain and bleeding will occur, the patient reflects a reasonable understanding about the plan for her surgery and the likely outcomes. Stating the doctor has nothing more to do so she is being released, stating that her surgeon indicated surgery is not necessary, and stating that she hasn't spoken to her surgeon all day indicate that the patient may be in denial and needs further explanation, instruction, and support before undergoing surgery.

You are the night shift nurse for a hospital nursing division of 40 acutely ill postoperative patients. The staffing for the night shift is you plus two patient care technicians. Based on the end-of-shift report, the current staffing, and your assessment of the patients, you have determined that there is insufficient staff to safely take care of the patients on this nursing division. What is the best action for you to take? A. Leave the nursing division immediately and go home. B. Contact the nursing supervisor, inform him or her of the situation, and leave the nursing division. C. Contact the chief of medicine and inform her or him of the situation and document it. D. Contact the nursing supervisor, inform him or her of the situation, and document it.

D Leaving the nursing division, even with documentation and notice of the situation to your nursing supervisor, may be seen as patient abandonment. Contacting the chief of medicine is insufficient notice to the nursing employer of the unsafe situation.

The nurse is caring for an older patient whose adult son is their healthcare proxy. The son is refusing to allow the mother to receive opioid medication for postoperative pain and to ambulate or engage in deep-breathing exercises because of pain. Which action by the nurse demonstrates ethical consideration for this patient? Asking a pain management specialist to speak to the son Requesting orders for the patient to receive non-narcotic and alternative treatments for pain Keeping the son's wishes confidential by logging out of the medical record after documenting them Understanding state law that the patient's son can refuse treatment

Requesting orders for the patient to receive non-narcotic and alternative treatments for pain

The nurse is documenting the care of a 16-year-old girl who has just discovered that she is pregnant. The patient is tearful as she reports that her boyfriend broke up with her 2 weeks ago and states that her life has truly been ruined. Which nursing diagnosis is appropriate in this situation? Self-Esteem, Situational Low, Risk for Noncompliance Social Isolation Health Behavior, Risk-Prone

Self-esteem, Situational Low, Risk for

A hospital that has a shortage of RNs but an increasing patient census is exploring ways to accommodate this increased patient volume without hiring additional RNs. The facility decides to increase the nurse/patient ratio and hire additional unlicensed assistive personnel (UAPs) to fill the gap. Which ethical dilemma does this solution present? This practice may discourage UAPs from furthering their careers because they already have steady employment. The hospital may not be able to attract qualified RNs if the nurse/patient ratio is increased. The decision could lead to a reduction in the quality of care provided. The hospital will be perceived as being cheap instead of cost-effective, resulting in bad publicity.

The decision could lead to a reduction in the quality of care provided.

a,c (Nurses should question the healthcare​ provider's orders that a client​ questions, and the healthcare​ provider's orders written before a​ client's condition changed. There is no need to question orders calling for medication with a decimal​ point, orders written after a​ client's condition​ improved, or orders for expensive​ in-house laboratory tests.)

Which healthcare provider​'s orders should nurses​ question, to protect themselves​ legally? ​(Select all that​ apply.) a Orders that a client questions b Orders written after​ client's condition improved c Orders written before​ client's condition changed d Orders for expensive​ in-house laboratory tests e Orders calling for a medication with a decimal point

A healthcare worker with an on-the-job back injury is placed on light duty. The worker is assigned to assist in the office with copying and filing, and asks the supervisor to move the desk closer to the copier due to pain while walking. The hurt employee provides supporting documentation from a physician, but the supervisor refuses to honor the request and terminates the worker for not performing the given responsibilities. The worker files a lawsuit for wrongful dismissal. Which act did this supervisor violate? a. American Disabilities Act b. Family Medical Leave Act c. Undue Hardship Leave Act d. Temporary Total Disability Act

a. American Disabilities Act

A supervisor requests to have only female nurses treat patients because the supervisor supports traditional gender roles. Why is this supervisor's request inappropriate? a. It violates the Civil Rights Act of 1964. b. It violates the Patient Protection and Affordable Care Act (PPACA). c. It violates the Occupational Safety and Health (OSH) Act of 1970. d. It violates the Emergency Medical Treatment and Active Labor Act (EMTALA).

a. It violates the Civil Rights Act of 1964.

A patient visits a primary care office with flu symptoms. During the appointment, the patient threatens the physician. As a result, the physician terminates the patient relationship. Why may this physician refuse to treat this patient? a. There is a safety concern. b. The staff feels uncomfortable. c. There is a minor medical complaint. d. The office is an independent practice.

a. There is a safety concern.

A nurse makes a significant medication error. Which information should the nurse expect the risk manager to ask​ about? A. The hours the nurse worked the previous day B. Any process that was in place that allowed the error to occur C. Whether the nurse intended to commit the error D. If the nurse will agree to have the error reported

b

The nurse suspects that a client is a victim of an assault. Which should be the​ nurse's priority​ action? A. Looking in the​ client's medical record to see if there is any history of assault or violence B. Notifying the supervisor and contacting police to report the injuries as suspicious C. Reporting the suspicion to risk management and asking them to investigate D. Documenting the findings and saying nothing to the other team members

b

A physician feels a patient does not have the ability to make a rational decision regarding termination of hemodialysis. However, the nurse knows the patient has an understanding of the choices regarding medical management based on conversations with the patient. The physician insists on disregarding the patient's pleas. Which patient right does this situation describe? a. Privacy b. Capacity c. Duty of care d. Advance directives

b. Capacity

A 35-year-old woman is suffering from a painful and aggressive stomach cancer. The woman is being cared for in the home by her husband. He has tried to make the wife more comfortable by giving pain medicine the way the hospice nurse outlined. Because the pain was unrelieved from the medicine, he quickly gave his wife another dose. The husband's intent was to relieve the wife's pain and not to cause death, but the wife died within the hour of respiratory failure. What is the result of the husband's dilemma? a. Euthanasia b. Double effect c. Assisted suicide d. Death with dignity

b. Double effect

A medical assistant discloses a patient's pregnancy test results to the patient's spouse, without consulting the physician. There is no authorization for disclosure form on file. For which two reasons is this disclosure unethical and what are the potential consequences? Choose 2 answers a. It violates the ACA and the medical assistant may be fired or fined. b. It violates HIPAA and the medical assistant may be fined or terminated. c. It violates veracity and the medical assistant's certification may be revoked. d. It violates the principle of role fidelity and the medical assistant may be terminated.

b. It violates HIPAA and the medical assistant may be fined or terminated. d. It violates the principle of role fidelity and the medical assistant may be terminated.

The National Association for Healthcare Quality (NAHQ) covers many principles that pertain to the ethical operation of an organization. Which standard addresses this concept? a. Supporting access to medical information by health providers b. Providing updated, relevant training to organizational leadership c. Establishing quality as the guiding principle for controlling costs d. Emphasizing programs that provide better patient reimbursement

b. Providing updated, relevant training to organizational leadership

An ethical dilemma ensued when a patient died in the emergency room. The patient had a driver's license that listed the patient as an organ donor, but the next of kin rejected organ donation. What is the next step? a. The hospital contacts the ethics committee to discuss next steps. b. The hospital harvests organs from this patient based on first person consent. c. The hospital contacts the regional organ procurement agency to decide next steps. d. The hospital does not harvest organs from this patient based on next of kin disagreement.

b. The hospital harvests organs from this patient based on first person consent.

Which action by the nurse correctly exemplifies the seven rights of medication​ administration? (Select all that​ apply.) A. Checking for the right frequency by looking at the​ client's chart B. Checking the prescribed order and looking at the time C. Checking for the right dose by performing a dose calculation and checking the medication D. Verifying the right medication by asking the​ client, "Is this what you normally take at​ home?" E. Documenting administration of the prescribed order in the client record

bce

Many provisions of the Patient Protection and Affordable Care Act (PPACA) were enacted to assist citizens in obtaining access to healthcare insurance and coverage. How does this act impact healthcare? a. Insurance companies must limit their premium charges to 85% of what they were charging prior to the implementation of the act on March 23, 2010. b. A provision for auto insurance in the act requires that auto insurance policies must cover usual and customary expenses associated with vehicular accidents. c. Group or individual health insurers providing dependent coverage of children must make coverage available to adult children up to age 26. d. Health insurers must offer part A, B, C, and D of Medicare to non-Medicare policy holders with a reasonable premium increase to cover the cost of this enhanced coverage.

c. Group or individual health insurers providing dependent coverage of children must make coverage available to adult children up to age 26.

A patient who is religious has an appointment with an ear, nose, and throat doctor to have his ears cleaned of wax buildup. When the patient arrives at the doctor's office, the nurse says his appointment is for another procedure the doctor determined to be necessary. After the appointment, the patient requested the doctor change the reason for the visit to increase reimbursement. Which aspect of a code of ethics did this physician violate? a. The patient will be provided with care of the highest quality. b. The patient's culture, religion, and heritage will be honored. c. The patient will be treated with honesty, dignity, and respect. d. The patient's right to execute advance directives will be followed.

c. The patient will be treated with honesty, dignity, and respect.

An 85-year-old patient is rushed to the hospital and diagnosed with a stroke. The physician advises the family that artificial nutrition and hydration will be required to keep the patient alive. The patient's family requests a consult with the hospital's ethics committee before proceeding with a decision. Which external issue would affect the ethics committee's advice to the family? a. The degree to which the patient is informed of risks b. The advance directive in place for the patient c. The various healthcare regulations that would apply d. The medical staff's comfort with taking timely action

c. The various healthcare regulations that would apply


Conjuntos de estudio relacionados

1. Islands of Indonesia and facts

View Set

Tumor Staging, Tumor Grade, and Tumor Markers

View Set

study set for quiz 2; graphing quadratic functions

View Set

Chapter 9: Multiple Regression-FIN 360

View Set

305 Financial Chapter 1 (Part 2)

View Set